You are on page 1of 81

INSPIRATION

SH. B.L. MORODIA


DEPUTY COMMISSIONER
KVS REGIONAL OFFICE JAIPUR

SH. D.R. MEENA


ASSISTANT COMMISSIONER
KVS REGIONAL OFFICE JAIPUR

Dr. LEKH RAM SAINI


PRINCIPAL
K.V. ALWAR
WORKSHOP COORDINATOR
संदेश

प्रियश्रीलेखरामसैनी,

मझ
ु े यह जानकर अत्यंत िसन्नता हो रही है कक केन्रीय प्रिद्यालय अलिर द्िारा कक्षा
दसिीं के प्रिज्ञान प्रिषय के प्रिद्यार्थियों हे तु निीन पाठ्यक्रम ि पैटनि आधाररत पाठ्य सामग्री
के ननमािण हे तु तीन ददिसीय संभागस्तरीय ऑनलाइन कायिशाला का आयोजन ककया जा रहा
है जो जयपुर संभाग के प्रिद्यालयों के प्रिज्ञान प्रिषय के स्नातकोत्तर/स्नातक शशक्षकों द्िारा
तैयार ककया जाना है आशा है कक उक्त कायिशाला में भाग लेकर सभी शशक्षक पूणि ननष्ठा और
लगन से अपेक्षक्षत पाठ्यसामग्री को तैयार करें गे जजसका अध्ययनकर कक्षा दसिीं के प्रिद्याथी
आगामी Term-I बोर्ि परीक्षा 2021-22 की तैयारी करने के ियोजन से अर्धक से अर्धक
लाभाजन्ित होंगे ।

मैं, उक्त पाठ्यसामग्री के ननमािण कायि से जुडे सभी शशक्षकों का प्रिशेषरूप से धन्यिाद
ज्ञापन करता हूूँएिं यह अपेक्षा करता हूूँ कक केन्रीय माध्यशमक शशक्षा बोर्ि के निीन पाठ्यक्रमि
पैटनि आधाररत पाठ्यसामग्री केननमािण से प्रिद्यार्थियों द्िारा उच्चस्तरीय अंक िाजतत का उद्दे श्य
अिश्य पूणि होगा ।

श्री लेखराम सैनी,


िाचायि,
केन्रीय प्रिद्यालय अलिर
STUDENT SUPPORT MATERIAL ( SAMPLE PAPERS)
CLASS –X
SCIENCE
SESSION 2021-22

MATERIAL PREPARATION COMMITTEE (TEAM WORK WITH TGT’s)

BANSHI DHAR KHATIK PGT CHEMISTRY KV 2 AJMER

AJAY SINGH SHAKTAWAT PGT PHYSICS KV BSF DABLA

VIDYADHAR SINGH PGT CHEMISTRY KV BSF JODHPUR

LALIT MOHAN CHATURVEDI PGT BIOLOGY KV KOTA NO.2


GOPAL SAIWLIYA PGT CHEMISTRY KV POKHRAN BSF

REVIEW COMMITTEE (TEAM WORK WITH TGT’s)

SEEMA KUMARI SHARMA PGT CHEMISTRY KV CHURU

MOHAN LAL SAINI PGT BIOLOGY KV LALGARH JATTAN

SUMAN TALWARA PGT CHEMISTRY KV JODHPUR NO.2 AFS

MAHIPAL SINGH PGT PHYSICS KV INDERPURA

DINESH KUMAR SONI PGT BIOLOGY KV INDERPURA

COMPILED BY:-
Mr.ANIL KUMAR ARYA
Mrs.DEEPTI GAUR
Mrs.SUMAN VERMA
(TGT SCIENCE)
KV NO.1 ALWAR
SCIENCE CLASS- X

COURSE STRUCTURE
EVALUATION SCHEME
THEORY
UNIT TERM-I MARKS
I CHEMICAL SUBSTANCES- NATURE AND BEHAVIOUR CH-1,2,3 16
II WORLD OF LIVING CH-6 10
III NATURAL PHENOMENA CH-10,11 14
UNITS TERM –II
I CHEMICAL SUBSTANCES- NATURE AND BEHAVIOUR CH-4,5 10
II WORLD OF LIVING CH-8,9 13
IV EFFECT OF CURRENT CH-12,13 12
V NATURAL PHENOMENA CH-15 05
TOTAL THEORY(TERM -I+TERM- II) 80
INTERNAL ASSESMENT: TERM –I 10
INTERNAL ASSESMENT: TERM- II 10
GRAND TOTAL 100
INDEX-TERM- I

S. NO. CHAPTER PERTICULARS


1 1 CHEMICAL REACTION AND EQUATIONS
2 2 ACIDS, BASES AND SALTS
3 3 METAL AND NON-METALS
4 6 LIFE PROCESSES
5 10 LIGHT (REFLECTION AND REFRACTION)
6 11 HUMAN EYE AND COLOURFUL WORLD
KENDRIYA VIDYALAYA SANGATHAN( RO JAIPUR)
Sample Question Paper -1 (Term-1) 2021-22
CLASS:X
Science (086)
Time :90 Minutes Max Marks: 40
General Instructions:
1.The Question Paper contains three sections.
2.Section A has 24 questions. Attempt any 20 questions.
3. Section B has 24 questions. Attempt any 20 questions.
4. Section C has 12 questions. Attempt any 10 questions.
5. All question carry equal marks .
6. There is no negative marking.
SECTION :A
Section –A contains of 24 questions. Attempt any 20 questions from this section.
The first attempted 20 questions would be evaluated.

1 Which of the following statements about the given reaction are correct?
3Fe(s) + 4H2O(g) → Fe3O4 (s) + 4H2 (g)
(i) Iron metal is getting oxidized
(ii) Water is getting reduced
(iii) Water is acting as reducing agent
(iv) Water is acting as oxidizing agent
(a) (i), (ii)and (iii) (b) (iii) and(iv)
(c) (i), (ii)and (iv) (d) (ii) and(iv)

2 Which of the following are exothermic processes?


(i)Reaction of water with quicklime
(ii)Dilution of an acids
(iii)Evaporation of water
(iv)Sublimation of camphor(crystals)
(a)(i)and(ii) (b) (ii) and(iii)
(c)(i)and(iv) (d) (iii) and(iv)
3 Three beakers labelled as A, B and C each containing 25 ml of water were taken. A
small amount of NaOH,anhydrous CuSO4 and NaCl were added to the beakers A, B and
C respectively. It was observed that there was an increase inthe temperature of the
solutions contained in beakers Aand B, whereas in case of beaker C, the temperature
of the solution falls. Which one of the following statement(s) is(are) correct?
(i) In beakers A and B, exothermic process has occurred.
(ii) In beakers A and B, endothermic process has occurred.
(iii) In beaker C exothermic process has occurred.
(iv) In beaker C endothermic process has occurred.
(a) (i)only (b) (ii)only
(c)(i)and(iv) (d) (ii) and(iii)

4 While performing an activity in the lab, four students added sodium sulphate solution to
four different solutions taken in four different test tubes. In which test tube is a white
precipitate observed after the reaction ?

(a)(i) (b) (ii)


(c)(iii) (d) (iv)

5 Electrolysis of water is a decomposition reaction. The mole ratio of


hydrogen and oxygen gases liberated during electrolysis of water is
(a) 1:1 (b) 2:1 (c) 4:1(d)1:2
6 Aluminium is used for making cooking utensils. Which of the following properties of
aluminium are responsible for the same?
(i) Good thermal conductivity
(ii) Good electrical conductivity
(iii) Ductility
(iv) High melting point
(a) (i) and (ii)
(b) (i) and (iii)
(c) (ii) and (iii)
(d) (i) and (iv)
7 While performing an activity in the lab students poured phenolphthalein indicator to four
different solutions taken in four different test tubes. A pink colour is observed in test tube
containing-

(a) HCl (b)NaOH (c) KCl (d) NaCl

8 Which of the given is used as an antacid?


(a) Sodium hydrogen carbonate
(b) Calcium hydroxide
(c) Magnesium hydroxide
(d) All the these

9 Sodium carbonate is a basic salt because it is a salt of


(a) strong acid and strong base
(b) weak acid and weak base
(c) strong acid and weak base
(d) weak acid and strong base
10 Sodium hydrogencarbonate when added to acetic acid evolves a gas. Which of the
following statements are true about the gas evolved?
(i) It turns lime water milky
(ii) It extinguishes a burning splinter
(iii) It dissolves in a solution of sodium hydroxide
(iv) It has a pungent odour
(a) (i) and (ii)
(b) (i), (ii) and (iii)
(c) (ii), (iii) and (iv)
(d) (i) and (iv)
11 Which statement is not correct about stomata?
a) Each stoma has a pair of guard cells
b) Stomata help to exchange of gases
c) Stamatas are tiny pores present in the leave
d) When water enters the guard cells, stomata closes

12 The mode of nutrition found in plant is:


a) Holozoic nutrition
b) Parasitic nutrition
c) Saprotrophic nutrition
d) Autotrophic nutrition

13 Glycolysis of respiration takes place in:


a) Cytoplasm
b) Mitochondria
c) Nucleus.
d) Mitochondrial matrix.
14 The correct path of urine in human excretory system is:
a) Kidney→ ureter → urethra→ urinary bladder
b) Kidney → urinary bladder → urethra → ureter
c) Kidney→ ureter → urinary bladder→ urethra
d) Urinary bladder → kidney → ureter →urethra

15 The cup-like hollow proximal part of each nephron is known as:


a) Loop of Henle
b) Bowman’s capsule
C) Collecting duct
D) capillary

16 Autotrophic organism converts of carbon dioxide and water into carbohydrates in the
presence of:
(a) carbon dioxide and sunlight
(b) chlorophyll and sunlight
(c) Carbon dioxide and Nitrogen
(d) Chlorophyll and carbon dioxide

17 Which one of the following materials cannot be used to make a lens?


(a) Water
(b) Glass
(c) Plastic
(d) Clay

18 No matter how far you stand from a mirror, your image appears erect. The mirror is likely
to be-
(a) Plane
(b) Concave
(c) Convex
(d) Either plane or convex

19 The angle between incident ray and reflected ray is 60c. What is the angle of incidence?
(a) 30c
(b) 40c
(c) 60c
(d) 50c
20 A convex lens has a focal length of 10 cm. At what distance from the lens should the object
be placed so that it forms a real and inverted image 20 cm away from the lens?
(a) -20 cm
(b) -40 cm
(c) -60 cm
(d) -80 cm

21 If an object is placed 10 cm in front of a concave mirror of focal length 20 cm, the image
will be :
(a) Diminished, upright, virtual
(b) Enlarged, upright, virtual
(c) Diminished, inverted, real
(d) Enlarged, upright, real
22
A white light falls on a glass prism. The least deviated colour is:
a violet
b orange
c yellow
d red

23 Rainbow is formed due to:


a internal reflection and dispersion of light through water droplet
b refraction, reflection and dispersion of light through a water droplet
c only dispersion of light
d only refraction of light

24 Length of the day is increased by 4minute dueto


(a) Scattering
(b) Refraction
(c) Total internal reflection
(d) Dispersion of light.
SECTION :B
Section –B contains of 24 questions. Attempt any 20 questions from this section.
The first attempted 20 questions would be evaluated.

25 Which of the following gases can be used for storage of fresh sample of an oil for a
long time?
(a) Carbon dioxide or oxygen
(b) Nitrogen or oxygen
(c) Carbon dioxide or helium
(d) Helium or nitrogen
26 Which of the following is(are) an endothermic process(es)?
(i) Dilution of Sulphuric acid(ii) Sublimation of dry ice
(iii) Condensation of water vapours(iv) Evaporation of water
(a)(i)and(iii)
(b) (ii)only
(c)(iii)only
(d) (ii) and(iv)

27 During the preparation of hydrogen chloride gas on a humid day, the gas is usually passed
through the guard tube containing calcium chloride. The role of calcium chloride taken in
the guard tube is to
(a) Absorb the evolved gas
(b) Moisten the gas
(c) Absorb moisture from the gas
(d) Absorb Cl-ions from the evolved gas
28 Common salt besides being used in kitchen can also be used as the raw material for
making
(i) Washing soda(ii) Bleaching powder
(iii) Baking soda(iv) Slaked lime
(a) (i) and (ii)
(b) (i), (ii) and (iv)
(c) (i) and (iii)
(d) (i), (iii) and (iv)

29 The pH of the gastric juices released during digestion is


(a) Less than 7 (b) More than 7
(c) Equal to 7 (d) Equal to 0
30 Which of the following pairs will give displacement reactions?
(a) FeSO4 solution and Copper metal
(b) AgNO3 solution and Copper metal
(c) CuSO4 solution and Silver metal
(d) NaCl solution and Copper metal

Question No. 31 to 34 consist of two statements – Assertion (A) and Reason (R). Answer
these questions selecting the appropriate option given below:

A. Both A and R are true and R is the correct explanation of A


B. Both A and R are true and R is not the correct explanation of A
C. A is true but R is false
D. A is False but R is true
31 Assertion: After whitewashing the walls, a shiny white colour on walls is obtained after 2
to 3 days.
Reason: Calcium hydroxide reacts with carbon dioxide to form calcium carbonate which
gives shiny white colour to the walls.
32 Assertion:There is no mixing of oxygenated and deoxygenated blood in the human heart .
Reason: Valves are present in the heart which allows movement of blood in one
direction only .
33 Assertion:Small intestine receives the secretion of the liver that digest fats.
Reason:Bile juice contains enzymes.

34 Assertion: Light travels faster in water than air.


Reason: Water is denser than air.
35 Which of the following oxide(s) of iron would be obtained on prolonged reaction of iron
with steam?
(a) FeO
(b) Fe2O3
(c) Fe3O4
(d) Fe2O3 and Fe2O
36 Which of following are not ionic compounds?
(i) KCl (ii) HCl (iii) CCl4 (iv) NaCl
(a) (i) and (ii)
(b) (ii) and (iii)
(c) (iii) and (iv)
(d) (i) and (iii)

37
The electronic configuration of three elements X, Y and Z are as follows:
X = 2, 4, Y = 2, 7, Z = 2,1 Which two elements will combine to form an ionic compound and
write the correct formula,
(a) X2Y
(b) ZY
(c) XZ3
(d)Y2Z
38 During photosynthesis the food is prepared in leaves, this food gets stored in the form of:
a) Glucose b) Fructose
c) Starch d) Glycogen
39 In case of kidney failure, an artificial kidney can be used for cleaning the blood of a person
by separating urea from blood is called:
(a) Osmosis
(b) Filtration
(c) Circulation
(d) Dialysis
40 Complete digestion of food takes place in:
(a) Small intestine
(b) Large intestine
(c) stomach
(d) Food pipe
41 The enzyme that break down starch into
simpler form is known as:
(a) Proteases
(b) Lipase
(c) Salivary amylase
(d) Trypsin
42 A boy is standing in front of a plane mirror at a distance of 3 m from it. What is the
distance between the boy and his image?
(a) 3 m
(b) 4.5 m
(c) 6 m
(d) None of these
43 The image formed by a concave mirror is real, inverted and of the same size as that of the
object. the position of the object should be :
(a) Beyond C
(b) Between C and F
(c) At C
(d) At F
44 Which type of mirror is used by ENT specialists as a ‘head mirror’?
(a) Plane mirror
(b) Convex mirror
(c) Concave mirror
(d) None of these

45 If a convex lens is used to focus sunlight on a paper, where the paper should be placed so
that it catches fire.
(a) At 25 cm away from lens
(b) At optical centre of lens
(c) At principal focus.
(d) At centre of curvature
46 When a ray of light passes through a glass prism . it suffers
a one refraction
b two refraction
c three refraction
d four reflection.
47 Water in deep sea is bluish in colour due to:
aabsorbtion of light by the sea
b reflection of sky in water
c scattering of light
d presence of plants in the sea.
48 Which of the following diagram correctly explain the passage of white light ?

SECTION :C
Section –Consists of three cases followed by questions. There are a total 12 questions in this
section . Attempt any 10 questions from this section.
The first attempted 10 questions would be evaluated.

Case Amoeba is an animal having no fixed shape.It ingests food particles by formation of
temporary finger‐like projections. The food vacoule inside amoeba breaks down
the food into small and soluble molecules.
The digested food is thrown out by the amoeba by the rupture of cell membrane and
it goes on for the search of next food particle.
49 How many nuclei an amoeba can have in their cell
a)1 b) 2 c) 3 d) 4
50 The amoeba is unicellular eukaryote that have ability to change its
a) Head b) Tail c) Shape d) Neck
51 In which of the following part ,the digestion of amoeba takes place
a)Endoplasm b) Ectoplasm c) Nucleus d) Food vacuole
52 What type of nutrition is followed by amoeba?
(a) Parasitic
(b) Holozoic
(c) Saprotrophic
(4) Autotrophic

Case The relationship between the distance of object from the lens (u), distance of image
from the lens (v) and the focal length (f) of the lens is called lens formula. It can be
written as 1/f=1/v-1/u. The size of image formed by a lens depends on the position of
the object from the lens. A lens of short focal length has more power whereas a lens of
long focal length has less power. When the lens is convex, the power is positive and
for concave lens, the power is negative.
The magnification produced by a lens is the ratio of height of image to the height of
object as the size of the image relative to the object is given by linear magnification
(m). When, m is negative, image formed is real and when m is positive, image formed
is virtual. If m < 1, size of image is smaller than the object. If m > 1, size of image is
larger than the object.

53 An object 4 cm in height is placed at a distance of 10 cm from a convex lens of focal


length 20 cm. The position of image is

a) 10 cm b) -10 cm c)20 cm d) -20 cm


54 In the above question, the size of image is
a)2 cm b)4 cm c) 6cm d)8 cm
55 An object is, placed 50 cm from a concave lens and produces a virtual image at a
distance of 10 cm in front of lens. The focal length of lens is

a)12 cm b) -12.5 cm c) 10 cm d) -10 cm

56 A concave lens of focal length 5 cm, the power of lens is


a) 0.2D b)-0.2 D c) -2D d) -20D
Case C.It is very interesting to note that our stomach produces hydrochloric acid. It helps
in the digestion of food without harming the stomach. During indigestion the stomach
produces too much acid and this causes pain and irritation. To get rid of this pain,
people use bases called antacids. These antacids neutralise the excess acid.
Magnesium hydroxide (Milk of magnesia), a mild base, is often used for this purpose.
57 PH of an acid is-
a) less than 7 b) more than 7 c) 7 d) zero
58 At what pH human body normally works
a) 7 to 7.8 b) 8to 9 c) 5to 7 d) 8 to 8.5
59 Name the chemical reaction which happens when stomach acid reacts with milk of
magnesia?
a) Addition reaction b) Neutralisation reaction c) Displacement reaction d) redox
reaction.
60 Which of the following is a strong base-
a) sodium hydroxide b)magnesium Hydroxide c) ammonium Hydroxide
d)aluminium Hydroxide

Question in lieu of diagram based question for VI - candidate .


Total alternate question 03.

4
Barium chloride on reacting with ammonium sulphate forms barium sulphate and
ammonium chloride. Which of the following correctly represents the type of the reaction
involved?

(i) Displacement reaction (ii) Precipitation reaction

(iii) Combination reaction (iv) Double displacement reaction

(a) (i) only (b) (ii) only

(c) (iv) only (d) (ii) and (iv)

7
Which of the following statements is correct about an aqueous solution of an acid and of a
base?

(i) Higher the pH, stronger the acid (ii) Higher the pH, weaker the acid

(iii) Lower the pH, stronger the base (iv) Lower the pH, weaker the base

(a) (i) and (iii) (b) (ii) and (iii)

(c) (i) and (iv) (d) (ii) and (iv)

48
What colour we obtain on mixing green and blue light?

a White

b Black

c Cyan

d Magenta
KENDRIYA VIDYALAYA SANGATHAN( RO JAIPUR)
Sample Question Paper -2 (Term-1) 2021-22
CLASS:X
Science (086)
Time :90 Minutes Max Marks: 40
General Instructions:
1.The Question Paper contains three sections.
2.Section A has 24 questions. Attempt any 20 questions.
3. Section B has 24 questions. Attempt any 20 questions.
4. Section C has 12 questions. Attempt any 10 questions.
5. All question carry equal marks .
6. There is no negative marking.
SECTION –A
Section-A consists of 24 questions. Attempt any 20 questions from this section.
The first attempted 20 questions would be evaluated.
1. Which of the following statement(s) s is/are true?
On exposure to sunlight for a long duration silver chloride turn Grey due to

(i) The formation of silver by decomposition of silver chloride


(ii) Decomposition of chlorine gas from silver chloride.
(iii) Sublimation of silver chloride
(iv) Oxidation of silver chloride
A.(i) Only
B.(i) and (ii)
C. (ii) and (iii)
D. (iv) and (ii)
2. In the refining of silver, the recovery of silver from silver nitrate solution involved
displacement of copper metal
2AgNO3 (aq) Cu (s) 2Ag (s) + Cu(NO3)2 (aq)
Which option in the given table correctly represents the substance reduced and
reducing agent?
Option Substance reduced Reducing agent

A AgNO3 Cu
B Ag Cu
C Cu AgNO3
D Cu Ag
3. Samples of metals A, B, C and D were taken and added to the following solutions
one by one. The results of obtained have been tabulated as follows.

Arrange the metals A, B, C and D in the order of decreasing reactivity:


A. B>D>A>C
B.A>B>C>D
C. D>C>A>B
D.. B>A>C>D
4. Which of the following are present in a dilute aqueous solution of hydrochloric
acid?
Option Positive ion Negative ion
A H3O+ OH¯
B H3O + Cl¯
C Cl+ OH¯
D OH+ Cl¯
5. In the electrolysis of water, name the gas which is collected in double volume than
the gas collected at anode

A. Oxygen
B.Hydrogen
C. Nitrogen
D. Helium
6. The Apparatus given in the adjoining figure was such as to demonstrate electrical
conductivity. Which of the following statement(s) is/are and correct.
A. The bulb will not glow because electrolyte is not acidic.
B. Bulb will not glow because it depends upon the type of electrolytic solution
C.Bulb will not glow because circuit is incomplete.
D.Bulb will glow because HCl is a strong acid and furnishes ions for conduction
7. Which of the following correctly represents a balanced chemical equation?
A. MnO2+ HCl(aq) MnCl2 + Cl2 + 2H2O
B. MnO2+ 2HCl (aq MnCl2 + Cl2 + 2H2O
C. MnO2+ 4HCl (aq MnCl2 + Cl2 + 2H2O
D. MnO2+ 4HCl(aq) MnCl2 + Cl2 + H2O
8. Equal volumes of hydrochloric acid and sodium hydroxide solutions of same
concentrations are mixed and the pH of resulting solution is checked with pH
paper. What would be the colour obtained?

A. Red
B. Yellow
C. Green
D. Blue
9. Rita was doing and activity, she observed that when crystals of copper sulphate are
strongly heated the the colour of of copper sulphate changes from Blue to White.
Why does so happen?
A. Due to photodecomposition of copper sulphate.
B. Due to loss of water molecules from Copper sulphate.
C. Due to oxidation of Copper sulphate.
D. Due to thermal decomposition of copper sulphate
10. There is an equation ‘X’, which contain equal number of atoms of each element on
both the sides. What is ‘X’?
A. A balanced equation.
B. An unbalanced equation.
C. A chemical equation.
D. Skeletal chemical equation
11. Carefully study the diagram of the human Circulatory system with labels 1, 3, 2 and
6. Select the option which gives correct identification and main function and /or
characteristic.

A (1) Pulmonary Artery: The pulmonary artery carries deoxygenated blood from
the right ventricle to the lungs.
B (3) Pulmonary Vein: The pulmonary vein carries deoxygenated blood from the
right ventricle to the lungs.
C (2) Superior Vena Cava: The superior vena cava carries oxygen rich blood from
the heart.
D (6) Left Ventricle: It supplies blood to the entire body and lungs.
12. Nephrons are made up of a cluster of thin walled capillaries called _____________
which is associated with a cup like structure called as __________ and the long tube
which terminates through this capsule.
A Glomerulus, Bowman’s capsule
B Neurons, Bowman’s capsule
C Nephrons, Bowman’s capsule
D Glomerulus, Convoluted tubule
13. The image shows the circulation of blood in fishes and human.

How is the circulation of blood in fish different from that in humans?


(a) The heart in fish is bigger in size.
(b) The flow of blood in fish is bidirectional.
(c) The blood goes through heart only once in fishes.
(d) The heart of fish has more chambers compared to that of human.

14. Given below is the figure of stomata. Select the correct labeling for the diagram.

(a) 1 Epidermal cell, 2 Stoma, 3 Guard cell, 4 Chloroplast


(b) 1 Guard cells, 2 Stoma, 3 Epidermal cells, 4 Chloroplast
(c) 1 Stoma, 2 Epidermal cells, 3 Chloroplast, 4 Guard cell
(d) 1 Chloroplast, 2 Stoma, 3 Epidermal cells, 4 Guard cell

15. The role of nasal cavity in Human Respiratory system is:


(i) Filtration of inhaled air
(ii) Removal of germs and dust
(iii) Moistening of inhaled air
(a) (i) and (ii)
(b) (ii) and (iii)
(c) (i), (ii) and (iii)
(d) None of these

16. In the experiment given here, water will rise in the tube because-

The correct reason to rise in the tube is-


(a) Oxygen of air in the flask will be taken up by the germinating seeds.
(b) Carbon dioxide given out by the germinating seeds will be absorbed by the
KOH.
(c) Carbon dioxide given out will go through the glass tube and will push water
up into the test tube.
(d) Moisture in the germinating seeds will reach the water in the beaker
through the delivery tube.

17. The properties of image formed by the plane mirror are:


(i) Virtual
(ii) Inverted
(iii) Laterally inverted
(iv)Size of the image is equal to that of the object.
Choose the correct option.
(a) (ii), (iii) and (iv)
(b) (i), (iii) and (iv)
(c) (i), (ii) and (iii)
(d) (i), (ii) and (iv)

18. A convex lens produces a magnification of +5. Where the object should be placed?
(a) At 2f
(b) Between 2f and f
(c) At f
(d) At less than f
19. Ram placed a glass slab on a drawing sheet and marked its boundaries PQRS. Then
he removed the glass slab and drew lines and angles as shown in the given figure.
He observed the path of light through the glass slab. Where does the incident and
emergent ray meet in this figure?
(a) Inside the glass
(b) Above the glass slab
(c) At infinity
(d) None of these
20. When an object is placed beyond the centre of the curvature(C) of a concave
mirror, the image formed is:
(a) Between the focus and the centre of the curvature(C)
(b) Real and inverted
(c) Smaller than the object
(d) All of the above
21. A student obtains a sharp image of the distant window (W) of the school
laboratory on the screen (S) using the given concave mirror (M) to determine its
focal length. Which of the following distances should he measure to get the focal
length of the mirror?

(a) MW
(b) MS
(c) SW
(d) MW- W
22. A beam of light is incident through the holes on side A and emerges out of the holes
on the other face of the box as show in the figure. Which of the following could be
inside the box?

(A)Concave lens
(B) Rectangular glass slab
(C) Prism
(D) Convex lens
23. Which of the following statements is true?
(a) A convex lens has 4 dioptre power having a focal length 0.25 m
(b) A convex lens has -4 dioptre power having a focal length 0.25 m
(c) A concave lens has 4 dioptre power having a focal length 0.25 m
(d) A concave lens has -4 dioptre power having a focal length 0.25 m.
24. A child is standing in front of a magic mirror. She finds the image of her head
bigger, the middle portion of her body of the same size and that of the legs smaller.
The following is the order of combinations for the magic mirror from the top.
(a) Plane, convex and concave
(b) Convex, concave and plane
(c) Concave, plane and convex
(d) Convex, plane and concave

SECTION –B
Section-B consists of 24 questions. Attempt any 20 questions from this section.
The first attempted 20 questions would be evaluated.
25 Which of these has pH less than 7?
(a) Copper Sulphate
(b) Sodium Bicarbonate
(c) Calcium Carbonate
(d) Magnesium Acetate
26. If a few drops of a concentrated acid accidentally spill over the hand of a student,
what should be done?
(a) Wash the hand with saline solution.
(b) Wash the hand immediately with plenty of water and apply a paste of sodium
hydrogen carbonate.
(c) After washing with plenty of water and apply solution of sodium hydroxide on
the hand.
(d) Neutralize the acid with a strong alkali.
27. Which of the following are exothermic processes?
(a) Reaction of water with quick lime
(b) Dilution of an acid
(c) Evaporation of water
(d) Sublimation of camphor (crystals)
A) (a) & (b)
B) (b) & (c)
C) (a) & (d)
D) (c) & (d)

28. Reaction between X and Y, forms compound Z. X loses electron and Y gains
electron. Which of the following properties is not shown by Z?
(a) Has high melting point
(b) Has low melting point
(c) Conducts electricity in molten state
(d) Occurs as solid

29 Increase in the OH– ion concentration, leads to


(a) An increase in the pH of solution
(b) A decrease in the pH of the solution
(c) Doesn’t alter the pH of the solution
(d) Decreases the basic strength of the solution
30. One of the constituents of baking powder is sodium hydrogen carbonate, the other
constituent is
(a) Hydrochloric acid
(b) Tartaric acid
(c) Acetic acid
(d) Sulphuric acid

Question No. 31 to 35 consist of two statements – Assertion (A) and Reason (R). Answer
these questions selecting the appropriate option given below:
A. Both A and R are true and R is the correct explanation of A
B. Both A and R are true and R is not the correct explanation of A
C. A is true but R is false
D. A is False but R is true
31. Assertion: The acid must always be added to water with constant stirring.
Reason: Mixing of an acid with water decreases the concentration of H+ ions per
unit volume.
32. Assertion:Copper is heated with the oxygen present in the air and forms a black-
coloured substance on its surface.
Reason: Black colour substance on copper surface is formed due to reduction of
copper into copper oxide.
33. Assertion: Turgor pressure is the positive pressure that develops in the plant cell
due to entry of water.
Reason: During growth of cells, the turgor pressure is responsible for enlargement
and extension.
34. Assertion: A ray of light travelling from a rarer medium to a denser medium slows
down and bends away from the normal. When it travels from a denser medium to
a rarer medium, it speeds up and bends towards the normal.
Reason: The speed of light is higher in a rarer medium than a denser medium.

35. Match the acids given in Column (A) with their correct source given in Column
Which of the following is correctly matched?

(A)Lactic acid-vinegar,Acetic acid-Lemon,Citric acid-Tomato,oxalic acid-curd


(B)Lactic acid-Lemon,Acetic acid-Tomato,Citric acid-curd,oxalic acid-vinegar
(C)Lactic acid-curd,Acetic acid-Lemon,Citric acid-Tomato,oxalic acid-vinegar
(D)Lactic acid-curd,Acetic acid-vinegar,Citric acid-lemon,oxalic acid-Tomato
36. What prevents the back-flow of blood inside the heart during contraction?
(A)thick muscular walls of ventricles
(B)valves
(C)thin walls of atria
(D)all of the above
37. 1. Which part of alimentary canal receives bile from the liver
(A)Stomach
(B)Small intestine
(C)Large intestine
(D)Esophagus
38. Transpiration and root pressure cause water to rise in plants by
(A)Pushing it upward
(B)Pushing and pulling it respectively
(C)Pulling and pushing it respectively
(D)Pulling it upward
39. Magnifying power of a concave lens is:
(A)always > 1
(B)always < 1
(C) always = 1
(D)can have any value
40. A 10 mm long awl pin is placed vertically in front of a concave mirror. A 5 mm long
image of the awl pin is formed at 30 cm in front of the mirror. The focal length of
this mirror is
(A) -30 cm
(B) -20 cm
(C) -40 cm
(D) -60 cm
41. The correct path of urine is:
a) Kidney→ ureter → urethra→ urinary bladder
b) Kidney → urinary bladder → urethra → ureter
c) Kidney→ ureter → urinary bladder→ urethra
d) Urinary bladder → kidney → ureter →urethra
42. What is the correct route for blood flow in a human?
a) Left atrium → Left ventricle → Lungs → Right ventricle → Right atrium
b) Left atrium → Left ventricle → Right ventricle → Right atrium → Lungs
c) Right atrium → Right ventricle → Left ventricle → Left atrium → Lungs
d) Right atrium → Right ventricle → Lungs → Left atrium → Left ventricle
43. The unit of power of lens is:
a) Meter
b) Diopter
c) Centimeter
d) M-1
44. A light ray enters from medium A to medium B as shown in figure. The refractive
index of medium B relative to A will be:

(a) greater than unity


(b) less than unity
(c) equal to unity
(d) zero

45. The optical phenomena, twinkling of stars, is due to:


(a) Atmospheric reflection
(b) Total reflection
(c) Atmospheric refraction
(d) Total refraction
46. The path of a ray of light coming from air passing through a rectangular glass slab
traced by four students are shown as A, B, C and D in figure. Which one of them is
correct?

(a) A
(b) B
(c) C
(d) D
47. Convex lens focus a real, point sized image at focus, the object is placed:

a) At infinity
b) Between F and 2F
c) At focus
d) At 2F
48. An element A is soft and can be cut with a knife. This is very reactive to air and
cannot be kept open in air. It reacts vigorously with water. Identify the element
from the following
a) Mg
b) Na
c) P
d) Ca
SECTION –C
Section-C consists of three cases followed by questions. There are a total of 12 questions in
this section. Attempt any 10 questions from this section.
The first attempted 10 questions would be evaluated.
Case

In the above image, an experiment is being performed where 2 g ferrous sulphate


crystals are taken in a dry boiling tube. The color of the ferrous sulphate crystals is
noted down. After that, the boiling tube is heated over the flame of a burner and
the colour of the substance is noted down. Based on the above experiment, answer
the questions given below.
49. What was the initial colour of the ferrous sulphate crystals?
(a) red
(b) white
(c) brown
(d) light green
50. What was the colour of the substance obtained on heating till decomposition.
(a) red
(b) white
(c) brown
(d) green
51. Which type of the reaction took place in boiling tube on heating?
(a) Decomposition reaction
(b) Precipitation reaction
(c) Displacement reaction
(d) Combination reaction
52. The substance obtained on heating ferrous sulphate crystals is
(a) ferric oxide
(b) Sulphur dioxide
(c) Sulphur trioxide
(d) all of the above
Case Shweta bought a potted rose plant from her nearby nursery. She loved her plant so
much that she kept the plant in the dark corner of her bedroom. After few days her
plant eventually started to dry though she provides water to her plant every day.
53. Which requirement is not fulfilled by the rose plant?

(a) water

(b) carbon dioxide

(c) sunlight

(d) chlorophyll
54. Which process got disturbed in the plant?
(a) photosynthesis

(b) breathing
(c) excretion
(d) reproduction

55. How the plant survived for a few days without performing photosynthesis?

(a) due to starch

(b) due to carbondioxide

(c) due to chlorophyll

(d) due to water


56. What would you suggest to Shweta for keeping her rose plant healthy?

(a) she should keep her plant in dark place

(b) the plant should have access to water, carbondioxide and sunlight

(c) she should transplant her rose in a bigger pot

(d) she should keep her plant in kitchen


Case If you have an aquarium or fish bowl at home, you might notice the fish looks
bigger when you look through the side. However, if you put your hand on the
opposite side of the aquarium/bowl, it also looks bigger.

57. Observe the picture and identify the reason?

(a) Due to reflection

(b) Due to refraction

(c) Due to dispersion

(d) Due to scattering


58. SI unit of refractive index is

(a) dioptre

(b) metre

(c) watt

(d) no unit
59. The formula to calculate the refractive index is

(a) n = cv

(b) n = v/c

(c) n = c/v

(d) v = nc
60. Glass has a refractive index ……… … than that of air.

(a) equal

(b) greater

(c) less

(d) impossible to say.


SAMPLE QUESTION PAPER (TERM I) 2020-21
CLASS X
ALTERNATIVE QUESTIONS IN LIEU OF DIAGRAM BASED QUESTIONS
1. Which of the following statement(s) s is/are true? On exposure to sunlight for a long
duration silver chloride turn Grey due to
(i) The formation of silver by decomposition of silver chloride
(ii) Decomposition of chlorine gas from silver chloride.
(iii) Sublimation of silver chloride
(iv) Oxidation of silver chloride
A.(i) Only
B.(i) and (ii)
C. (ii) and (iii)
D. (iv) and (ii)
5. Name the gas which is collected in double volume than the gas collected at anode in
electrolysis of water.
A. Oxygen
B. Hydrogen
C. Nitrogen
D. Helium
6. Which solution does not conduct electricity:
A) Aqueous solution of NaCl
B) Aqueous solution of NaOH
C) Aqueous solution of Glucose.
D) Solution of Lemon juice
9. Rita was doing and activity, she observed that when crystals of copper sulphate are
strongly heated the colour of of copper sulphate changes from Blue to White. Why
does so happen?
A. Due to photo decomposition of copper sulphate.
B. Due to loss of water molecules from copper sulphate.
C. Due to oxidation of copper sulphate.
D. Due to thermal decomposition of copper sulphate
11. Which is the correct sequence of air passage during inhalation
A. Nostrils-larynx-pharynx- trachea- lungs
B. Nostrils- pharynx- larynx- trachea-alveoli
C. Nostrils- pharynx- larynx- alveoli -trachea
D. Nostrils- pharynx- larynx- alveoli-bronchi
12. Identify the correctly match option:
Name of enzyme Site of secretion
A. Amylase i. small intestine
B. Pepsin ii. Wall of Esophagus
C. Trypsin iii. Pancreas
D. Lipase iv. Large intestine

13. Which of following activity essential for opening and closing of stomata in guard cell: -
A. Respiration
B. Excretion
C. Photosynthesis
D. Transpiration

16. Choose the correct statement that describes arteries


2. A. They have thick elastic walls, blood flows under high pressure; collect
3. blood from different organs and bring it back to the heart
4. B.They have thin walls with valves inside, blood flows under high pressure and
5. carry blood away from the heart to various organs of the body
6. C. They have thick elastic walls, blood flows under low pressure; carry blood
7. from the heart to various organs of the body
8. D. They have thick elastic walls without valves inside. The Blood flows
9. under high pressure and carry blood away from the heart to different parts of the
body
19. Rainbow is caused due to
A.) Reflection of light from Air.
B.) Dispersion of Sun light from water drops.
C)Refraction of sun light from water drops.
D)Diffraction of sun light from water drops.
21. The image of an object placed in front of a convex mirror is formed at: -
A.) At object distance itself.
B.) Twice the distance in front of mirror.
C.) Half the distance in front of mirror.
D.) Behind the mirror.
22. What is the refractive index of the medium if angle of incidence is equal to angle of
refraction?
A.)1.5
B.)1
C.)0.5
D)2
44. There is an equiconvex lens of focal length 20 cm. If the lens is cut into two equal parts
perpendicular to principal axis then focal length of each lens is:-
A.)10 cm
B.)40 cm
C.)15 cm
D.)20 cm
46. A concave lens has a focal length of 10 cm. The object of height 2 mm is placed at a
distance of 15 cm from the pole. Find the height & nature of image.
A.)3.5 cm & Inverted
B.)4.0 cm & Erect
C.)3.0 cm & Virtual
D.)3.5 cm & Erect

CASE STUDY BASED QUESTIONS


Case Chemical Reaction
An experiment is performed where 2 g ferrous sulphate crystals are taken in a dry
boiling tube. The color of the ferrous sulphate crystals is noted down. After that, the
boiling tube is heated over the flame of a burner and the colour of the substance is
noted down. Based on the above experiment, answer the questions given below.
49. What was the initial colour of the ferrous sulphate crystals?
(a) red
(b) white
(c) brown
(d) light green
50. What was the colour of the substance obtained on heating till decomposition.
(a) red
(b) white
(c) brown
(d) green
51. Which type of the reaction took place in boiling tube on heating?
(a) Decomposition reaction
(b) Precipitation reaction
(c) Displacement reaction
(d) Combination reaction
52. The substance obtained on heating ferrous sulphate crystals is
(a) ferric oxide
(b) Sulphur dioxide
(c) Sulphur trioxide
(d) all of the above
Case Atmospheric refraction:-
If you have an aquarium or fish bowl at home, you might notice the fish looks bigger
when you look through the side. However, if you put your hand on the opposite side of
the aquarium/bowl, it also looks bigger.
Answer the question given below on the basis of above given passage.
57. According to you what may be the reason?

(a) Due to reflection

(b) Due to refraction

(c) Due to dispersion

(d) Due to scattering


58. SI unit of refractive index is

(a) dioptre

(b) metre

(c) watt

(d) no unit
59. The formula to calculate the refractive index is

(a) n = cv

(b) n = v/c

(c) n = c/v

(d) v = nc
60. Glass has a refractive index ……… … than that of air.

(a) equal

(b) greater

(c) less

(d) impossible to say.


KENDRIYA VIDYALAYA SANGATHAN( RO JAIPUR)
Sample Question Paper -3 (Term-1) 2021-22
CLASS:X
Science (086)
Time :90 Minutes Max Marks: 40
General Instructions:
1.The Question Paper contains three sections.
2.Section A has 24 questions. Attempt any 20 questions.
3. Section B has 24 questions. Attempt any 20 questions.
4. Section C has 12 questions. Attempt any 10 questions.
5. All question carry equal marks .
6. There is no negative marking.

SECTION - A
Section – A consists of 24 questions. Attempt any 20 questions from this section. The first
attempted 20 questions would be evaluated.
1 X + Barium chloride →Y(s ) White ppt + Sodium chloride
Here X and Y are:
(a) NaSO4, BaSO4
(b) BaSO4, Na2SO4
(c) BaSO2, NaSO4
(d) Na2SO4, BaSO4
2 Which one of the following pair is correct
Reaction Reaction Type
(a) 2KNO3(s) → 2KNO2(s) +O2( g) Displacementreaction
(b) Zn(s ) +2AgNO3→ Zn(NO3 )2+2Ag(s) Combinationreaction
(c) Ni( NO3)2 (aq) +2NaOH→ Ni(OH)2 Double displacement
+2NaNO3 (aq)↓ reaction and
precipitation reaction
(d) N2+ 3H2→ 2NH3(g) Decomposition reaction
3 The following reaction is an example of
4 NH3( g)+5O2( g)→ 4NO(g) +4H2O(g)
1. Displacement reaction
2. Combination reaction
3. Redox reaction
4. Neutralization reaction
(a) 1 and 4
(b) 2 and 3
(c) 1 and 3
(d) 3 and 4

4 Which of the following is not a balanced equation?


(a) Ca(OH)2+ CO2 → CaCO3 + H2O
(b) Fe + CuSO4→ FeSO4+ Cu
(c) KClO4 → KCl+ 2O2
(d) Cu + 2HNO3 →Cu(NO3)2 + 2NO2 + H2O

5 When the gases sulphur dioxide and hydrogen sulphide mix in the presence of
water, the following reaction takes places: SO2 + 2H2S → 2H2O + 3S
Here, hydrogen sulphide is acting as:
(a) an oxidizing agent
(b) a reducing agent
(c) a dehydrating agent
(d) a catalyst

6 On burning magnesium ribbon in air, it is observed that it burns brightly leaving


behind a .......... powder.
(a) White
(b) Green
(c) Yellow
(d) Black

7 Which of the following gives the correct increasing order of acid strength ?
(a) Water < acetic acid < hydrochloric acid
(b) Water < hydrochloric acid < acetic acid
(c) Acetic acid < water < hydrochloric acid
(d) Hydrochloric acid < water < acetic acid

8 When NaOH and HCl are mixed in equal molar quantities , the result is –
(a) The formation of salt + water ( H2O)
(b) The formation of salt + hydrogen ( H2)
(c) The formation of salt + oxygen ( O2)
(d) All above re correct

9 An element ‘X’ forms two oxides ‘XO’ and ‘ XO2’ . The oxide ‘XO’ is neutral but ‘
XO2’ is acidic in nature . The element ‘X’ is –
(a) Sulphur
(b) Calcium
(c) Hydrogen
(d) Carbon

10 Aluminium is used for making cooking utensils. Which of the following properties
of Aluminium are responsible for the same?
(i) Good thermal conductivity
(ii) Good electrical conductivity
(iii) Ductility
(iv) High melting point
(a) (i) and (ii)
(b) (i) and (iii)
(c) (ii) and (iii)
(d) (i) and (iv
11 The breakdown of pyruvate to give carbon dioxide, water and energy takes place in
(a) Mitochondria (c) nucleus
(b) Cytoplasm (d) Chloroplast

12 If salivary amylase is lacking in the saliva, which of the following events in


the mouth cavity will be affected?
(a) Proteins breaking down into amino acids
(b) Starch breaking down into sugars
(c) Fats breaking down into fatty acids and glycerol
(d) Absorption of vitamins

13 Identify the micro-organism whose nutrition type is shown below :

(a) Yeast (c) Bacteria


(b) Amoeba (d) Paramecium

14 The function of bile is to ….


(a) Synthesis proteins (c) Breakdown proteins
(b) Emulsify fats (d)Breakdown
carbohydrate

15 The function of sphincter muscle is to


(a) Improves digestion (c) Regulate digestive process
(b) Release digestive juices (d) Regulate exit of food

16 If stomata is blocked, then which of the following processes will not occur?
(a) Respiration and photosynthesis
(b) Transpiration, photosynthesis and respiration
(c) Transpiration and respiration
(d) Respiration, transpiration and transportation

17 Light rays A and B fall on optical component X and come out as C and D.

The optical component is a –


(a) concave lens
(b) convex lens
(c) prism
(d) none of these
18 The image shows the path of incident rays to a concave mirror. Where would the
reflected rays meet for the image formation to take place?

(a) behind the mirror


(b) between F and O
(c) between C and F
(d) beyond C

19 The image shows reflection of light on a mirror. Based on the image, what can be
inferred?

(a) The incident ray, reflected ray, and normal at the point of reflection lie on a
common plane
(b) The angle of incidence, angle of reflection, and normal at the point of
reflection lie on a common plane.
(c) The angle between incident ray and normal is greater than the angle between
normal and the reflected ray.
(d) The angle between incident ray and normal is smaller than the angle
between normal and the reflected ray

20 The image shows the path of light travelling through a glass slab. What causes the
ray of light to deviate from its original path?
(a) change in the amount of light
(b) change in the direction of wind flow
(c) change in the temperature of the air
(d) change in the density of the medium

21 A student conducts an experiment using a convex lens. He places the object at a


distance of 60 cm in front of the lens and observed that the image is formed at a
distance of 30 cm behind the lens. What is the power of the lens?
(a) 0.005 dioptre
(b) 0.05 dioptre
(c) 5 dioptre
(d) 50 diptre

22 When a beam of white light falls on a glass prism, the colour of light which will
deviate least is:-
(a) Violet (b) Red (c) Green (d) Blue

23 Blue color of the sky is due to the phenomenon of:

(a) Reflection of light (b) Refraction of light

(c) Dispersion of light (d) Scattering of light

24 The star appear shifted from their actual position due to the phenomenon of:

(a) Diffraction of light (b) Scattering of light

(c) Refraction of light (d) Reflection of light

SECTION – B
Section – B consists of 24 questions (SI. No.25 to 48 ) . Attempt any 20 questions from
this section. The first attempted 20 questions would be evaluated.
25 Which of the following are exothermic processes?
(i) Reaction of water with quick lime
(ii) Dilution of an acid
(iii) Evaporation of water
(iv) Sublimation of camphor (crystals)
(a) (i) and (ii)
(b) (ii) and (iii)
(c) (i) and (iv)
(d) (ii) and (iv)
26 pH paper is separately dipped into two different solutions X and Y . Colour of pH
paper turned pale green in X and blue in Y. X and Y are most probably –
(a) X – H2O , Y – NaOH
(b) X – NaOH , Y – H2O
(c) X – HCl , Y – NaOH
(d) X – NaOH , Y – HCl

27 Which one of the following metal oxides shows both acidic and basic characters ?
(a) Na2O
(b) K2O
(c) CuO
(d) Al2O3

28 The formula of Gypsum and Plaster of Paris is –


(a) CaSO4.2H2O and CaSO4.1 ½ H2O
(b) CaSO4.2H2O and CaSO4.1/2 H2O
(c) CaSO4 and CaSO4.1/2 H2O
(d) CaSO4.2H2O and CaSO4.3H2O

29 Which of the following salt does not contain the water of crystallisation ?
(a) Blue vitriol
(b) Baking soda
(c) Washing soda
(d) Gypsum

30 The poorest conductor of heat among metals is


(a) Lead
(b) Copper
(c) Calcium
(d) Sodium

Question No. 31 to 35 consist of two statements – Assertion (A) and Reason (R). Answer
these questions selecting the appropriate option given below:
(a) Both A and R are true and R is the correct explanation of A
(b) Both A and R are true and R is not the correct explanation of A
(c) A is true but R is false
(d) A is False but R is true
31 Assertion : The aqueous solutions of glucose and alcohol do not show acidic
character .
Reason : Aqueous solutions of glucose and alcohol do not give H+ ions

32 Assertion : The process of dissolving an acid or a base in water is highly


exothermic reaction.
Reason :Water must always be added slowly to acid with constant stirring .

33 Assertion :Silver chloride turns grey is sunlight.


Reason :Silver is one of the least reactive metals.

34 Assertion: Walls of ventricles are thicker than the auricles.


Reason : This helps in preventing the back flow of the blood.

35 Assertion: The Sun looks reddish during sunrise and sunset.


Reason: The sun is directly overhead at noon and the sunlight has to travel relatively
shorter distance

36 Al2O3 + 2NaOH → …… + H2O


(a) Al(OH)3
(b) Na2O
(c) NaAlO2
(d) AlNaO2
37 In humans, the oxygen rich blood from the lungs come to the :
(a) Left ventricle (c) Left atrium
(b) Right ventricle (d) Right atrium

38 The inner lining of the .......... has numerous finger-like projections called villi
which increase the surface area for absorption.
(a) Stomach (c) Pancreas
(b) Small intestine (d) Large intestine

39 The role of nasal cavity in human respiratory system

(i) Removal of germs and dust. (iii) Filtration of inhaled air.


(ii) Moistening of the inhaled air.
(a) (i) & (ii) (c) (i) & (iii)
(b) (i),(ii) & (iii) (d) None of these

40 Artificial removal of nitrogenous wastes from the human body in the event of
kidney failure is
(a) Plasmolysis
(b) Dialysis
(c) Diffusion
(d) Osmosis

41 The function of trypsin is to


(a) Digest fats .
(b) Digest protein.
(c) Digest starch .
(d) Digest sugar .

42 An object of height 10 cm is placed in front of a convex lens having focal length of 12


cm. The object is placed at a distance of 36 cm in front of the lens. How many times is
the image likely to be magnified?
(a) 0.5 times
(b) 2 times
(c) 3 times
(d) 4 times

43 The image represents the rays of light travelling through a concave lens.
Where is the image most likely to form?
(a) P
(b) Q
(c) R
(d) S

44 You are given water, mustard oil, glycerine and kerosene. In which of these media a
ray of light incident obliquely at same angle would bend the most?
A. Kerosene

B. Water
C. Mustard oil
D. Glycerine

45 Which image represents the path of incident and reflected ray from a convex mirror
when an object is placed at infinity?

A. B.

D.
C.

(a) A
(b) B
(c) C
(d) D

46 The magnification of a spherical mirror is ± 3 . Then the mirror must be


a) Plane
(b) Concave
(c) Convex
(d) Any one of these
47 A concave mirror can form a real and enlarged image of an object when
the object is placed
(a) at the centre of curvature
(b) at a distance less than its focal length
(c) at a distance greater than its radius of curvature
(d) between focus and centre of curvature

48 In which of the following media, the velocity of the light will be maximum
(a) glass
(b) water
(c) diamond
(d) vacuum

SECTION – C
Section- C consists of three Cases followed by questions. There are a total of 12 questions in
this section. Attempt any 10 questions from this section.
The first attempted 10 questions would be evaluated.

Case Bleaching powder is also known as chloride of lime . It is a solid and yellowish
white in colour . Bleaching powder can be easily identified by the strong smell of
chlorine . When calcium hydroxide ( slaked lime ) reacts with chlorine , it gives
calcium oxy-chloride ( bleaching powder ) and water is formed . Aqueous
solution of bleaching powder is basic in nature . The material to be bleached is
first passed through solution of NaOH to remove greasy matter . Then it is passed
through aqueous solution of bleaching powder and very dil. HCl solution .HCl
reacts with bleaching powder to liberate nascent oxygen which bleaches material
.

49 Bleaching powder is used as –


(a) Bleaching agent in textile, paper and jute industry
(b) Disinfectant for water to make water free of germs
(c) Oxidising agent in many industries
(d) All of these

50 Bleaching powder is also known as –


(a) Calcium oxy-chloride
(b) Calcium hypochlorite
(c) Chloride of lime
(d) All of these

51 Bleaching powder gives smell of chlorine because it –


(a) Is unstable
(b) Gives chlorine on exposure to atmosphere
(c) Is a mixture of chlorine and slaked lime
(d) Contains excess of chlorine

52 Identify the product ‘X’ in the given reaction –


Ca(OH)2 + Cl2 X + H 2O
(a) CaOCl2
(b) CaCl2
(c) Ca(ClO3)2
(d) CaCO3

Case The given diagram represents the structure of human excretory system. Study
the diagram and answer the questions that follows:-

53 Identify the part 1 in excretory system .


(a) Kidney (c) Ureter
(b) Urethra (d) Nephron

54 Which of these is the structural and functional unit of part 2?


(a) Alveoli (c) Nephron
(b) Neuron (d) None of these

55 How blood is purified by artificial method/s?


(a) Filtration (c) Dialysis
(b) Re-absorption (d) All of these

56 The excretory materials are temporarily stored in:


(a) Urethra
(b) Kidneys
(c) Ureters
(d) Urinary bladder
Case A child was drawing ray diagrams to locate the position of images when an
object is placed at various locations in front of a lens. By mistake, he after
completing his drawings kept the pencil in the glass filled with water and was
amused by what he saw.
57 What amused the child?
(a) The color of water.
(b) The bending of pencil.
(c) He kept pencil in glass instead of pencil stand.
(d) The reaction between pencil and glass.
58 In vacuum the speed of light depends upon
(a) Frequency
(b) Wavelength
(c) Velocity of the source of light
(d) None of these
59 As light travels from a rarer to a denser
medium it will have
(a) increased velocity
(b) decreased velocity
(c) decreased wavelength
(d) both (b) and (c)
60 The speed of light in air is 3×108 m s–1, whereas that of the speed of light in water
is 2.26×108 m s–1. What is the refractive index of water with respect to air?
(a) 1
(b) 0.75
(c) 1.32
(d) 2.64
KENDRIYA VIDYALAYA SANGATHAN( RO JAIPUR)
Sample Question Paper -4 (Term-1) 2021-22
CLASS:X
Science (086)
Time :90 Minutes Max Marks: 40
General Instructions:
1.The Question Paper contains three sections.
2.Section A has 24 questions. Attempt any 20 questions.
3. Section B has 24 questions. Attempt any 20 questions.
4. Section C has 12 questions. Attempt any 10 questions.
5. All question carry equal marks .
6. There is no negative marking.

SECTION – A

It consists of 24 questions. Attempt any 20 questions from this section. The first
attempted 20 questions would be evaluated.

1. Water undergoes decomposition to produce hydrogen and oxygen due to


(a) Heat
(b) Sunlight
(c) Chemical reaction
(d) Electric current
2. A student writes a chemical equation of the reaction between quicklime and water.
CaO +H2O → Ca (OH) 2
How can the reaction be classified?

(a) The reaction is an example of a combination reaction as a compound separates into


two compounds.
(b) The reaction is an example of a decomposition reaction as a compound dissociates
into two compounds.
(c) The reaction is an example of a combination reaction as two compounds react to
form a single compound.
(d) The reaction is an example of a decomposition reaction as two compounds react to
form a single compound.
3. Identify the nature of substance formed in the following activity:

(a) Basic because it turns red litmus to blue


(b) Acidic because it turns red litmus to blue
(c) Neutral
(d) None of these
4. Non-metal which is liquid at room temperature is:
(a)Mercury (b) Bromine (c)Carbon (d) helium
5. Number of protons in an atom of an element A is 19 then, the number of electron in its ion
A+ is:
(a) 18 (b)19 (c)20 (d)21
6. Nettle sting is natural source of which acid
(a) Methanoic acid (b) Lactic acid (c) Citric acid (d)Tartaric acid
7. A solution reacts with crushed egg-shells to give a gas that turns lime-water milky. The
solution contains:
(a) NaCl (b) HCl (c) LiCl (d) KCl
8. Which one of the following is formed when calcium hydroxide reacts with carbon dioxide?
(a) Hydrogen gas (b) Water (c) Salt (d) Both (b) and (c)
9. If 10ml of H2SO4 is mixed with 10ml of Mg(OH)2 of the same concentration, the resultant
solution will give the following colour with universal indicator:
(a) Red (b) Yellow (c) Green (d) Blue
10. Which of the following salts does not contain water of crystallization?
(a) Blue vitriol (b) Baking soda (c) Washing soda (d) Gypsum
11. The diagram given below is the experimental set-up to show that carbon dioxide is given
out during respiration. In this set-up what does the test tube marked (X) contain?

(a) Water (b) Lime water (c) Potassium hydroxide (d) Hydrochloric acid

12. The main function of the part C is to

(a) control the pressure of urine in the urinary bladder


(b) take the urine from the kidney to the urinary bladder
(c) filter blood and remove the urine
(d) connect the parts of the excretory system

13. The opening and closing of stomata pore depends upon ___.
(a) Oxygen concentration in plants
(b) Guard Cells
(c) Concentration of carbon dioxide in stomata
(d) Temperature
14. Which of the following option is incorrectly match
Enzyme Function
(a) Pepsin Digest protein in stomach
(b) Trypsin Digest protein in small intestine
(c) Lipase Digest fats into fatty acids and glycerol
(d) Amylase Emulsify fats

15.Mammalian lungs are


(a) solid
(b) Spongy
(c) Solid and spongy
(d) Hollow and spongy

16. Left atrium of human heart contains:

(a) Oxygenated blood


(b) Deoxygenated blood
(c) Mixed blood
(d) Pure blood

17. The S.I unit of Power of a lens is _________.


(a) cm (b) m (c) cm-1 (d) Dioptre
18. Light from the sun falling on a convex lens will converge at a point called …
(a) Centre of curvature (b) Focus (c) Optical centre (d) Radius of curvature
19. A convex lens form same size image when object placed at :-
(a) At F (b) At 2F (c) Between F and 2F (d) Between F and optical centre (o)
20. What is the unit of refractive index:-
(a) Dioptre (b) Degree (c) unit less (d) m/sec
21. A light ray enters from medium A to medium B as shown in figure. The refractive index of
medium B relative to A will be

(a) Greater than unity (b) less than unity (c) equal to unity (d) zero
22. As light travels from rare to denser medium it’s velocity will
(a) Increases velocity (b) Decreases velocity
(c) No Change in velocity (d) None of the above
23. Power of a lens is +2D find out its focal length :
(a) 0.5cm (b) – 0.5cm (c) 50cm (d) -50 cm

24. Twinkling of stars is due to atmospheric


(a) AtmosphericDispersion of light by water droplets
(b) AtmosphericRefraction of light by different layers of varying refractive indices
(c)Atmosphericscattering of light by dust particles
(d)AtmosphericInternal reflection of light by clouds
SECTION - B
It consists of 24 questions (Sl. No.25 to 48). Attempt any 20 questions from this
section. The first attempted 20 questions would be evaluated.
25. Before burning in air, the magnesium ribbon is cleaned by rubbing with a sand paper to:
( a) Make the ribbon surface shinier
( b) Remove the layer of magnesium oxide from the ribbon surface
(c) Remove the layer of magnesium carbonate from the ribbon surface
(d) Remove the moisture from the ribbon surface.

26. One of the following processes does not involve a chemical reaction. That is:
( a) Melting of candle wax when heated
( b) Burning of candle wax when heated
(c) Digestion of food in our stomach
(d) making of compost from vegetable waste.

27. Which of the following represent the correct order of decreasing reactivity?

(a)Mg > Zn > Fe


(b) Mg > Zn > Al > Fe
(c)Al > Zn > Fe > Mg
(d) Mg > Fe > Zn > Al
28. Element reacts with oxygen to give a compound with a high melting point. This compound
is also soluble in water. The compound is likely to be
(a) CaO (b) CO 2 (c) SiO2 (d)FeO
29. What happens when a pellet of sodium is dropped in water?

(a) It absorbs heat and forms oxide.


(b) It catches fire and forms hydroxide.
(c) It absorbs heat and forms hydroxide.
(d) It catch heat and forms oxide.
30. What is the value of refraction angle when light incident vertically enter in a transparent
medium:-

(a) 0° (b) 30° (c) 90° (d) 60°

Given below are two statements labelled as Assertion (A) and Reason (R)

(a)Both assertion (A) and reason (R) are true and reason (R) is the correct explanation
of assertion.
(b)Both assertion (A) and reason (R) are true but reason (R) is not the correct
explanation of assertion.
(c) Assertion (A) is true but reason (R) is false.
(d) Assertion (A) is false but reason (R) is true.
31. Assertion (A): exposure of silver chloride to sunlight for a long duration turns grey due
to the formation of silver by decomposition of silver chloride
Reason (R): in this process sublimation of silver chloride takes place
32. Assertion (A) :- When copper oxide is added to dilute hydrochloric acid, the colour of the
solution becomes blue-green.
Reason (R) :- Copper(II) Chloride has blue green colour.
33. Assertion (A): Muscles of stomach wall possess thick layers of muscle fibres.
Reason (R): These muscles help in mixing the food with the enzymes present in the
alimentary canal.
34. Assertion: - Focal length of a plane mirror is infinity.
Reason: Focal length depend on the curvature of mirror
35. Which of the following pairs of reactants will go undergo a displacement reaction?
(a)CuSO4 + Fe (b)ZnSO4 + Fe (c)MgSO4 + Fe (d)Ca(SO4)2 + Fe

36. The life process that remove toxic substance of an organism:

(a)Respiration (b) Nutrition (c) Transportation (d) Excretion


37. Only two of the following Statements accurately describe what happens in the mouth

(a)Amylase breaks down large starch molecules into smaller maltose molecules.

(b)Chewing increases the surface area of food for digestion.

(c ) Saliva emulsifies fats into smaller droplets.

(d )Teeth breakup large insoluble molecules into smaller soluble molecules.

Which statements are correct?

(a) 1 and 2 (b) 2 and 3 (c) 3 and 4 (d) 1 and 4


38. Transportation of organic food in plants is called
(a) Translocation
(b) Transpiration
(c) Photosynthesis
(d) Transduction
39. If the magnification of a mirror is m = - (1/3 ) . Name the mirror.
( a) Concave mirror ( b) Convex mirror
(c) Both concave mirror and convex mirror (d) None of the above

40. Which of the following statements is true?

(a) A convex lens has 4 dioptre powers having a focal length -0.25 m

(b) A convex lens has -4 dioptre power having a focal length 0.25 m

(c) A concave lens has 4 dioptre power having a focal length 0.25 m

(d) A concave lens has -4 dioptre power having a focal length -0.25 m
41. The substance used to test starch:

(a) Copper sulphate (b) Alcohol (c) Iodine (d) Amylase


42. The absorbed food reaches to all cells of body through blood. The use of digested food is:

(a) Produce energy by respiration (b) Used to make fat

(c) Used to make protein (d) All of these

43. A ray diagram of refraction through prism is shown. Locate the angle of prism and angle
of deviation, respectively
(a) 9,1

(b) 5,8

(c) 1,9

(d) 9,8

44. The sky appears dark to passengers flying at very high altitude mainly because-

(a) Scattering of light is not enough at such heights

(b) There is no atmosphere at great heights

(c) The size of molecules is smaller than the wavelength of visible light

(d) The light gets scattered towards the earth


45. At sun rise or at sun set the sun appears to be reddish while at mid-day it looks white. This
is because

(a) Scattering due to dust particles and air molecules causes this phenomenon

(b) The sun is cooler at sun rise or at sunset

(c) Refraction causes this phenomenon

(d) Diffraction sends red rays to the earth at these times

46. The splitting of white light into several colours on passing through a glass prism is due to

(a) Refraction

(b) Reflection

(c) Interference

(d) Dispersion

47. The following one is not a primary colour

(a) purple

(b) Red

(c) Green
(d) Blue

48. The danger signals installed at the top of tall buildings are red in colour. These can be
easily seen from a distance because among all other colours, the red light-
(a) Is scattered the most by smoke or fog

(b) Is scattered the least by smoke or fog

(c) Is absorbed the most by smoke or fog

(d) Moves fastest in air

SECTION – C

It consists of three Cases followed by questions. There are a total of 12 questions in


this section. Attempt any 10 questions from this section. The first attempted 10
questions would be evaluated.

Read the following and answer the following questions:

Ionic compound is a chemical compound in which ions are held together by ionic bonds. An
ionic bond is the type of chemical bond in which two oppositely charged ions are held through
electrostatic forces. We know that, metal atoms have loosely bound valence electrons in their
valence shell and non-metal atoms need electrons in their valence shell to attain noble gas
configuration. The metal atom loses the valence electrons while non-metal atom accepts these
electrons. By losing electrons, metal atoms change to cations and by accepting electrons, non-
metals form anions. Ionic compounds are generally solid and exist in the form of crystal. They
have high melting and boiling points.

49. Which of the following can change to a cation?


(a) Fluorine

(b) Oxygen

(c) Potassium

(d) Neon

50. Which of the following can change to an anion?


(a) Iodine

(b) Magnesium

(c) Calcium

(d) Xenon
51. Ionic compounds are soluble in _____________.
(a) Kerosene

(b) Petrol

(c) Water

(d) None of these


52. Select the incorrect statement.
(a) Ionic compounds are generally brittle

(b) Ions are the fundamental units of ionic compounds


(c) Formation of ionic bonds involves sharing of electrons.

(d) NaCl is an ionic compound.


Read the following and answer the following questions:

Lymphatic system consists of lymph, lymph capillaries, lymph vessels, lymph nodes. Fluid
diffuses through the very thin walls of the capillaries into the tissue spaces. It contains
minerals, nutrients and fewer proteins and nourishes the tissues. This tissue fluid is called
lymph. The lymphatic system helps to transport this tissue fluid back into the main blood
stream, From intercellular spaces, lymph goes into lymphatic capillaries. Lymphatic capillaries
join to form large lymph vessels that drain into collecting ducts. These empty the lymph into
the two subclavian veins, located below the collar bones. These veins join to form the superior
vena cava. Lymph flows only in direction that is from tissues to heart through veins.

53. Identify the correct feature of lymph?


(a) It is similar to the plasma of blood, but is colourless and contains fewer proteins.
(b) It is similar to the WBCs of blood, but is colourless and contains more proteins.
(c) It is similar to the RBCs of blood and red in colour.
(d) It contains more fats.
54. Lymph flows in

(a) intercellular spaces

(b) extracellular space

(c) in arteries

(d) in veins

55. The important function of lymph is

(a) return RBCs to lymph nodes

(b) return interstitial fluid to blood

(c) carry oxygen

(d) carry waste and undigested food

56. Which of the following is not a main function of lymph glands

(a) forming RBCs

(b) Forming WBCs

(c) Forming antibodies

(d) Destroying bacteria

Read the following and answer the following questions:

Rohan conduct an experiment to find the position and nature of image form by a convex
lens. He find out that convex lens always converge the incident ray. The upper and lower
edge of this lens is thinner compare to its middle portion so the degree of convergence is
different when light incident on different portion of the lens. Rohan use the lens formula to
find the image position of the object.
57. A convex lens has a focal length of 15 cm. At which of the following position should an
object be placed so that this convex lens form erect and magnify image?
(a) 18 cm (b) 10 cm (c) 20 cm (d) 25 cm

58. Nature of images formed by a convex lens are always:-


(a) Real, inverted and small (b) Virtual, erect and large
(c) Real, inverted and large (d) None of these

59. A small bulb is placed at the focal point of a convex (converging) lens. When the bulb is
switched on, the lens produces:
(a) A parallel beam of light (b) A divergent beam of light
(c) A convergent beam of light (d) A patch of colored light

60. The focal length of four convex lenses P, Q, R, and S are 20cm, 15 cm, 5 cm and 10 cm,
respectively. The lens having greatest power is:
(a) P (b) Q (c) R (d) S

Questions in lieu of diagram based questions for VI candidates

Total Alternative Questions -04

3. Identify the nature of substance formed in burning of Magnesium

(a) Basic because it turns red litmus to blue

(b) Acidic because it turns red litmus to blue

(c) Neutral

(d) None of these

11. Experimental set-up to show that carbon dioxide is given out during respiration, what is
the material in tube?

(a) Water (b) Lime water (c) Potassium hydroxide (d) Hydrochloric acid

21. A light ray enters from medium A to medium B,light bends towards the normal.The
refractive index of medium B relative to A will be

(a) Greater than unity (b) less than unity (c) equal to unity (d) zero
43. A ray diagram of refraction through prism, observation are follows. Locate the angle of
prism and angle of deviation, respectively

Angle between produced incident ray and 9


produced transmitted ray

Angle formed at the vertex of prism 1

Refracted angle 3

Angle opposite to the prism angle 5

(a)9,1 (b)5,8 (c) 1,9 (d) 9,8


KENDRIYA VIDYALAYA SANGATHAN( RO JAIPUR)
Sample Question Paper -5 (Term-1) 2021-22
CLASS:X
Science (086)
Time :90 Minutes Max Marks: 40
General Instructions:
1.The Question Paper contains three sections.
2.Section A has 24 questions. Attempt any 20 questions.
3. Section B has 24 questions. Attempt any 20 questions.
4. Section C has 12 questions. Attempt any 10 questions.
5. All question carry equal marks .
6. There is no negative marking.
SECTION – A
Section – A consists of 24 questions. Attempt any 20 questions from this section. The first
attempted 20 questions would be evaluated .

1. Rahul took 5ml of Sodium sulphate solution in a beaker and added approximately 4ml of
Barium Chloride solution to it. A white ppt is formed. What is the chemical name of
white compound?
A. Barium sulphate B. Sodium phosphate
C. Sodium chloride D. Barium nitrate

2. Identify gas X in the following experiment.

Which gas is passed in lime water?


A. Hydrogen B. Nitrogen
C. Carbon dioxide D. Oxygen
3. Ions are present in Ionic compounds in-
A. Solid state B. Molten state
C. Solution form D. A, B and C
4. A solution of slaked lime is used for white washing walls. Calcium hydroxide reacts
slowly with the carbon dioxide in air to form a thinlayer of calcium carbonate on the
walls. After two tothree days of white washing it gives a shiny finish to the walls.
Compound which is formed after 2-3 days on walls –
A. CaCl2 B. CaO
C. CaCO3 D. CaSO4
5. The graph given below depicts a neutralization reaction.

Identify letter which denotes the area of the graph where only salt present
A) A B) B C) C D) D

6. Identify X in the reaction given

A ) Dark B). Light C). Heat D). Electric spark

7. Combustion is considered as an
A. Exothermic reaction B. Endothermic reaction reaction
C. oxidation reaction D. Reduction reaction
8. Strength of acids is
A. proportional to pH B. inversely proportional to pH
C. not related to pH D. None
9. Name the process while using mild bases like baking soda to protect from the pain and
irritation caused by stung of honey-bee?
A. Precipitation B. Sedimentation
C. Neutralization D. Oxidation

10. The surface of copper powder becomes black coated. This is due to formation of-
A. Copper oxide B. Copper carbonate
C. Copper chloride D. Copper sulphide
11. The rate of breathing in aquatic organisms is much faster than that seen in terrestrial
organisms, because-
A. The amount of dissolved oxygen in water is fairly high as compared to the amount of
oxygen in the air.
B. The amount of dissolved oxygen in water is fairly low as compared to the amount of
oxygen in the air.
C. Aquatic organisms need more oxygen to breath.
D. Aquatic organisms do not have proper

12. Identify the option that indicates the correct enzyme that is secreted in location q
mouth (buccal cavity), stomach and small intestine.

Options Mouth Stomach Small intestine

(buccal cavity)

A Lipase trypsin pepsin

B Amylase pepsin trypsin

C Trypsin amylase lipase

D Lipase amylase pepsin


13.
Which statement is true regarding opening and closing of stomata
A. Water entry in guard cells causes stomata to open.
B.Water exit from the guard cells causes stomata to close.
C. BothAand B are correct.
D. No effect of water on opening and closing of stomata.
14.Which of the following statement(s) is (are) true about the heart?
(i) The left atrium receives oxygenated blood from different parts of the body while the
right atrium receives deoxygenated blood from lungs.
(ii) Left ventricle pumps oxygenated blood to different body parts while right ventricle
pumps deoxygenated blood to lungs.
(iii) Left atrium transfers oxygenated blood to the right ventricle which sends it to
different body par
(iv) The right atrium receives deoxygenated blood from different parts
of the body while the left ventricle pumps oxygenated blood to
different parts of the body.
A.(i) B. (ii) C. (ii) and(iv) D. (i) and (iii)
15. Observe the diagram and identify correct pair: -
A. A-Vein, B-Vein, C-Artery, D-Artery
B. A-Vein, B-Artery, C-Vein, D-Artery
C. A-Artery, B-Vein, C-Artery, D-Vein
D. A-Vein, B-Artery, C-Artery, D-Vein

16. Which of the following events in the mouth cavity will be affected if salivary amylase is
lacking in the saliva?
A. Starch breaking down into sugars
B. Proteins breaking down into amino acids
C. Absorption of vitamins
D. Fats breaking down into fatty acids and glycerol

17. A full length image of a distant tall building can definitely be seen by using
A. a concave mirror B. a convex mirror
C. a plane mirror D. both concave as well as plane mirror

18. A lens produces an enlarged, virtual image, what kind of lens is it?
A. Diverging B. Converging
C. it could be either diverging or converging D. none of these

19. The laws of reflection hold good for


A. plane mirror only B. concave mirror only
C. convex mirror only D. all mirrors irrespective of their shape

20. Light travel fastest in


A. Water B. Air
C. Glass D. Diamond

21. The radius of curvature of a mirror is 20cm the focal length is


A. 20cm B. 10cm C. 40cm D. 5cm

22. An object is placed 60cm in front of a convex mirror. The virtual image formed by the
mirror is located 30 cm behind the mirror. What is the object's magnification?
A. +2 B. -2 C. -0.5 D. +0.5
23. When light passes from into glass it experiences change of -
A. Speed only B. wavelength and speed
C. frequency only D. frequency and speed

24. In a glass prism -


A. blue light is dispersed more than red light
B. red light dispersed more than blue light
C. both red light and blue light are equally
dispersed
D. None of these
SECTION – B
Section - B consists of 24 questions (Sl. No.25 to 48). Attempt any 20 questions from this
section. The first attempted 20 questions would be evaluated.

25. The respiration process during which glucose undergoes slow combustion by combining
with oxygen in the cells of our body to produce energy, is a kind of:
a.Exothermic process
b. Endothermic process
c. Reversible process
d. Physical process
26. The figure given below represents the experiment carried out between conc. sulphuric
acid and sodium chloride, which react with each other to form HCl gas.

Blue litmus paper is brought near the mouth of the delivery tube to check the presence
of HCl acid but no change is observed in the color of litmus paper because:
A ).The litmus paper used is dry
B). The litmus paper used is moist
C ) Blue litmus paper does not change its color with an acid
D). The litmus paper is kept very close to the mouth of the delivery tube

27. Which of the following is the correct arrangement of the given metals in order of their
reactivity?
Zinc, Iron, Magnesium, Sodium
(a) Zinc > Iron > Magnesium > Sodium
(b) Sodium > Magnesium > Iron > Zinc
(c) Sodium > Zinc > Magnesium > Iron
(d) Sodium > Magnesium > Zinc > Iron
28. The ability of metals to be drawn into thin wire is known as
(a) Ductility (b) Malleability (c) Sonority (d) Conductivity
29. What happens when a solution of an acid is mixed with a solution of a base in a test
tube?
(i) Temperature of the solution decreases
(ii) Temperature of the solution increases
(in) Temperature of the solution remains the same
(iv) Salt formation takes place
(a) (i) and (iv)
(b) (i) and (iii)
(c) (ii) only
(d) (ii) and (iv)
30. The apparatus given in the adjoining figure was set up to demonstrate electrical
conductivity.

Which of the following statement(s) is (are) correct?


(i) Bulb will not glow because electrolyte is not acidic.
(ii) Bulb will glow because NaOH is a strong base and furnishes ions for conduction.
(iii) Bulb will not glow because circuit is incomplete.
(iv) Bulb will not glow because it depends upon the type of electrolytic solution.

(a) (i) and (iii) (b) (ii) and (iv) (c) (ii) only (d) (iv) only

Question No. 31 to 35 consist of two statements – Assertion (A) and Reason (R).
Answer these questions selecting the appropriate option given below:
A. Both A and R are true and R is the correct explanation of A
B. Both A and R are true and R is not the correct explanation of A
C. A is true but R is false
D. A is False but R is true
31. Assertion : When zinc is added to dilute hydrochloric acid, hydrogen is given
off.
Reason : Hydrogen chloride molecules contain chlorine and hydrogen atoms.
32. Assertion : Photosynthesis is considered as an endothermic reaction.
Reason : Energy gets released in the process of photosynthesis.
33. Assertion : Valves are present in the arteries.
Reason : Arteries carry oxygenated blood from heart to different body parts
except for the pulmonary artery.
34. Assertion : A white light on passing through a prism splits into its component
colours as such that the red light emerges nearest to the base of the prism.
Reason : Wavelength of red light is more than other component colours and
hence, red light deviates least.
35. Which of the following statements is correct about an aqueous solution of an acid and of
a base?
(i) Higher the pH, stronger the acid
(ii) Higher the pH, weaker the acid
(iii) Lower the pH, stronger the base
(iv) Lower the pH, weaker the base

(a) (i) and (iii) (b) (ii) and (iii) (c) (i) and (iv) (d) (ii) and (iv)

36. Single circulation, i.e., blood flows through the heart only once during one cycle of
passage through the body, is exhibited by :
(A) Labeo, Chameleon, Salamander (B) Hippocampus, Exocoetus, Anabas
(C) Hyla, Rana, Draco (D) Whale, Dolphin, Turtle

37. Choose the correct path of oxygen in our body :


(A) Nostril →Trachea→ Bronchioles→ Alveoli
(B) Lungs → Trachea → oesophagus → Alveoli
(C) Nostril → oesophagus →Bronchioles →Alveoli
(D) Bronchioles →Trachea → Nostril → Alveoli
38. Transpiration helps in
a) Temperature regulation
b) Maintaining upward movement of water
c) Wilting of leaves
d) Both a and b
39. Rays from the sun converge at a point 15 cm in front of a concave mirror. Where should
an object be placed so that the size of its image is equal to the size of the object?
(A) 15 cm in front of the mirror
(B) 30 cm in front of the mirror
(C) Between 15 cm and 30 cm in front of the mirror
(D) More than 30 cm in front of the mirror
40. Magnification produced by a rear-view mirror fitted in
vehicles
(A) is less than one.
(B) is more than one.
(C) is equal to one.
(D) can be more than or less than one depending upon the position of the objectin front
of it.

41. In which mode of nutrition an organism de-rives its food from the body of another living
organism without killing it?
(a) Saprotrophic nutrition (b) Parasitic nutrition
(c) Holozoic nutrition (d) Autotrophic nutrition.
42. Which is the correct sequence of body parts in the human alimentary canal?
(a) Mouth → stomach → small intestine → large intestine →oesophagus
(b) Mouth →oesophagus→ stomach → small intestine → large intestine
(c) Mouth → stomach →oesophagus → small intestine → large intestine
(d) Mouth → oesophagus → stomach → large intestine → small intestine
43. The image shows the path of incident rays to a concave mirror.

Where would the reflected rays meet for the image formation to take place?

(a) behind the mirror (b) between F and O

(c) between C and F (d) beyond C

44. A student studies that the speed of light in air is 300000 kms/ sec where that of speed in
a glass slab is about 197000 kms/ sec. What causes the difference in speed of light in
these two media?

(a) difference in optical density (b) difference in temperature

(c) difference in amount of light (d) difference in direction of wind flow

45. Lateral inversion is shown by-

(a) Plane mirror

(b) Convex mirror

(c) concave mirror

(d) All
46. Image formed by a convex spherical mirror is:

(a) virtual (b) real (c) enlarged (d) inverted

47. A lens of focal length 12 cm forms an erect image, three times the size of the object. The
distance between the object and image is:

(a) 8 cm (b) 16 cm (c) 24 cm (d) 36 cm

48. Which property of metals is used for making bells and strings of musical instruments
like Sitar and Violin?
(a) Sonorousness (b) Malleability (c) Ductility (d) Conductivity

Section C
Section- C consists of three Cases followed by questions. There are a total of 12 questions in this
section. Attempt any 10 questions from this section. The first attempted 10 questions would be
evaluated.

Case study 1
pH is quite useful to us in a number of ways in daily life. Some of its applications are:
Control of pH of the soil : Plants need a specific pH range for proper growth. The soil may be
acidic, basic or neutral depending upon the relative concentration of H+ and OH-. The pH of any
soil can be determined by using pH paper. If the soil is too acidic, it can be corrected by adding
lime to it. If the soil is too basic, it can be corrected by adding organic manure which contains
acidic materials.
Self-defence by animals through chemical warfare : Stings of bees and ants contain
methanoic acid. When stung, it causes lot of pain and irritation. This can be cured by rubbing
the affected area with mild base like baking soda.
49. P is an aqueous solution of acid and Q is an aqueous solution of base. When these
two are diluted separately, then
a. pH of P increases while that of Q decreases.
b. pH of P decreases while that of Q increases.
c. pH of both P and Q decrease.
d. pH of both P and Q increase.
50. Which of the following acids is present in bee sting?
(a) Formic acid (b) Acetic acid (c) Citric acid (d) Hydrochloric acid
51. Sting of ant can be cured by rubbing the affected area with soap because
(a) it contains oxalic acid which neutralises the effect of formic acid
(b) it contains aluminium hydroxide which neutralises the effect of formic acid
(c) it contains sodium hydroxide which neutralizes the effect of formic acid
(d) none of these
52. The pH of soil X is 7.5 while that of soil Y is 4.5. Which of the two soils, should be treated
with powdered chalk to adjust its pH?
(a) X only (b) Y only (c) Both X and Y (d) none of these

CASE STUDY 2

They create by-products that are not only unclean for the cells of the body, but could even be
harmful. These waste by-products are therefore needed to be removed from the body and
disordered outline by a process called excretion. Again if the basic rules for body design in
multi-cellular organisms are followed, a specialized tissue for excretion will be developed ,
which means that the transportation system, will need to transport waste away from cells to
this excretory tissue.
53. The Urinary bladderdo-
(a) Produce urine (b) Carry urine (c) Temporarily store urine (d)All
54. The main excretory by-product in human beings in
(a) Creatine (b) Urea (c) Uric acid (d) None of the above
55. The process of removal of nitrogenous waste materials from the body is called -------
(a) Nutrition (b) Respiration (c) Excretion (d) Transportation
56. Which is main excretory organ in human beings.
(a) Intestine (b) Kidneys (c) Lungs (d) Heart

CASE STUDY 3
A concave lens in thick at the edges and thin at the centre, while a convex lens is thick at the
centre and thin at the edges. We can distinguish between a concave lens and a convex lens
without touching them. For this keep a book close to a lens and observe the image of the text of
the book through the lens. If the letters appear diminished then it is a concave lens.
Convex lens converges light rays and hence known as converging lens. Similarly concave lens
diverged light rays and is known as diverging lens. Linear magnification produced by a lens is
equal to the ratio of the image distance to the object distance. Power of a lens is defined as the
reciprocal of its focal length.
57. A concave lens is thin at the
(a) Edges
(b) Centre
(c) Both centre and edges
(d) None of them
58. Which of the following is converging lens
(a) Convex (b) Concave (c) Both A and B (d) None of them
59. The power of convex lens taken as
(a) Positive (b) Negative (c) Both A and B (d) None of them
60. The power of concave lens of focal length of 2m in
(a) 0.5 D (b) – 0.5 D (c) 1 D (d) - 1 D

Questions in lieu of diagram based questions for VI candidates


Total alternative questions -26

SECTION – A
2. Which change occur in lime water(Calcium Hydroxide) after passing the carbon
dioxide gas?
A. Turn blue B. Turn white C. Turn Black D. No change
3. Which of the following non-metal is lustrous?
A.Sulphur B. Oxygen C. Nitrogen D. Iodine
5. Na2CO3.10H2O is known as-
A Baking Soda
B. Baking Powder
C.Washing Soda
D. Bleaching Powder
11. In amoeba, food is digested in the:
A. food vacuole
B. mitochondria
C. pseudopodia
D. chloroplast
12 When a few drops of iodine solution are added to rice water, the solution turns
blue- black in colour. This indicates that rice water contains:
A. fats
B. complex proteins
C. starch
D. simple proteins
14. The back flow of blood during the pumping of blood by heart is prevented by:
A .Walls of ventricles
B. Walls of atrium
C. Valves in heart
D.Walls between atria and ventricles
15. In which part of the alimentary canal food is finally digested?
A. Stomach
B. Mouth cavity
C. Large intestine
D. Small intestine
16. Lack of oxygen in muscles often leads to cramps among cricketers. This results
due to
A.conversion of pyruvate to ethanol
B.conversion of pyruvate to glucose
C. non conversion of glucose to pyruvate
D.conversion of pyruvate to lactic acid
18. Which kind of mirror is used in rear view mirror?
A.Plane mirror
B.Convex mirror
C. Concave mirror
D.none of these
22. Identify correctlens magnification formula-
A. Height of image÷ Height of object
B .Height of Object÷ Height of image
C. Distance between object and image÷ 100
D. Height of image÷ Height of object X 100
23. Wavelength and speedof light changes when -
A. it enters into prism
B. it enters into glass slab
C. it enters into water
D. All
24. On dispersion through a glass prism which color is not produced -
A. Blue, Red, Green
B. Blue, Red, Orange
C. Blue, Pink, Green
D. None of these

Section B
26. Which one of the following salts does not contain water of crystallization?
(a) Blue vitriol
(b) Baking soda
(c) Washing soda
(d) Gypsum
27. What happens when calcium is treated with water?
(i) It does not react with water
(ii) It reacts violently with water
(iii) It reacts less violently with water
(iv) Bubbles of hydrogen gas formed stick to the surface of calcium
(a) (i) and (iv)
(b) (ii) and (iii)
(c) (i) and (ii)
(d) (iii) and (iv)
30. Which of the following metals occur in their pure state?
(a)Copper
(b)Iron
(c)Zinc
(d)Gold
40. A Virtual image is formed by a concave mirror when the object is placed between
(a) infinitive and centre of curvature
(b) centre of curvature and focus
(c) Between focus and pole
(d) All of the above
43. If refractive indices for water and diamond relative to air are 1.33 and 2.4
respectively, then the refractive index of diamond related to water is -
(a) 0.55
(b) 1
(c) 3.19
(d) 1.80
45. Identify kind of lens used in microscope-
(a) Concave
(b) Convex
(c) Bifocal
(d) plane glass
SECTION –C
Case study 1
Heterotrophic nutrition is a mode of nutrition in which organisms obtain readymade organic
food from outside sources. The organisms that depend upon outside sources for obtaining
organic nutrients are called heterotrophs.
Heterotrophic nutrition is of three types: saprophytic, parasitic and holozoic nutrition.
53. In which of the following groups of organism’s food material is broken outside the
body and absorbed?
(a) Mushroom, green plants, Amoeba
(b) Yeast, mushroom, bread mould
(c) Paramecium, Amoeba, Cuscuta
(d) Cuscuta, lice, tapeworm
54. Which of the following is a parasite?
(a) Yeast
(b) Taenia
(c) Amoeba
(d) Earthworm
55. Which of the following is an example of saprotroph?
(a) Grass
(b) Mushroom
(c) Amoeba
(d) Paramecium
56. Heterotrophic nutrition involves
(a) production of simple sugar from inorganic compounds
(b) utilization of chemical energy to prepare food
(c) Utilization of energy obtained by plants
(d) all of these.
Case Study 2
Ajay has seen that white light is dispersed into its seven colour component by a prism.
Different colors of light paint through different angles with respect to the incident as
the passed through a prism the red light bends the least while the violet the most. Discovery of
each color image along different path and thus become distinct. It is the band of distinct color
that we see in a spectrum.
57. White light is passed through a prism, the color which Deviate least is
(a) Violet
(b) Red
(c) Blue
(d) Green
58. The following one is not a primary color
(a) Green
(b) Red
(c) Blue
(d) Yellow
59. Orange, blue and yellow are three of the colours formed by a prism. Their older
according to increasing deviation is
(a) blue, Orange, yellow
(b) Yellow, blue, orange
(c) Blue, yellow, orange
(d) Orange, yellow, Blue
60. Dispersion is the term used to describe
(a) The propagation of light in straight lines.
(b) The splitting of a beam of light into component colours.
(c) The bending of a beam of light when it strikes a mirror.
(d) The change that takes place in white light after passage through red glass.
{MARKING SCHEME}
KENDRIYA VIDYALAYA SANGATHAN( RO JAIPUR)
MARKING SCHEME – SQP 1
CLASS:X
Science (086)
ANSWER KEY: SECTION -A
1. (c) (i), (ii)and (iv)
2.(a)(i)and (ii)
3.(c)(i)and (iv)
4.(a) (i) Na2SO4 + BaCl2 → BaSO4 +2NaCl
5.(b) 2:1
6. (d) (i) and (iv)
7. (b) NaOH,sodium hydroxide
8 . (d) All the these
9.(d) weak acid and strong base
10.(b) (i), (ii) and (iii)
11. d , When water enters the guard cells, stomata closes
12.d Autotrophic nutrition
13. a. Cytoplasm
14. c. Kidney→ ureter → urinary bladder→ urethra
15. b. Bowman’s capsule
16. b chlorophyll and sunlight
17. (d) Clay
18 (d) Either plane or convex
19.(a) 30c
20. (a) -20 cm
21. (b) Enlarged, upright, virtual
22. d red
23. b reflection, refraction and dispersion of white light through a water droplet
24. b refraction.
SECTION –B
25. (d) Helium or Nitrogen
26.(d) (ii) and(iv)
27 .(c) Absorb moisture from the gas
28.(c) (i) and (iii)
29. (a) less than 7
30. (b) AgNO3 solution and Copper metal
31. A, Both A and R are true and R is the correct explanation of A
32. B. Both A and R are true and R is not the correct explanation of A.Double circulatory system
is correct explanation .
33. C. A is true but R is false.
34. D , A is False but R is true.
35. (c) Fe3O4
36. (b) (ii) and (iii) .KCl& NaCl ionic
37. (b) ZY
38. C. Starch
39. D, Dialysis
40. A. Small intestine
41. C, Salivary amylase
42. (c) 6 m
43. (c) At C
44. (c) Concave mirror
45. (c) At principal focus.
46. b two refraction
47. c scattering of light
48.a.
SECTION :C
CASE A 49.a. one nucleus
50.c, Shape
51.d Food vacuole
52.b , Holozoic
B 53.d, -20 cm
54.d, 8 cm
55.b , -12.5 cm
56.d , -20D
C 57.a, less than 7
158.a, 7 to 7.8
59.b ,Neutralisation reaction
60.a, sodium hydroxide.

ANSWER KEY:
Diagram based question for VI - candidate .
Total alternate question 03.

Q4. (d) (ii) and (iv)

Q.7. (d) (ii) and (iv)

Q.48. (a) white


KENDRIYA VIDYALAYA SANGATHAN( RO JAIPUR)
MARKING SCHEME – SQP 2
CLASS:X
Science (086)

Q.NO ANSWERS (SECTION-A)


1. A . (i) Only
2.
A AgNO3 Cu
3. D .. B>A>C>D
4. B H3O+ Cl¯
5. B Hydrogen
6. D .Bulb will glow because HCl is a strong acid and furnishes ions for conduction

7. C . MnO2+ 4HCl(aq)---------->MnCl2 + Cl2 +2H2O


8. C Green
9. B Due to loss of water molecules from Copper sulphate.
10. A balanced equation
11. Option A

12. Option A Glomerulus, Bowman’s capsule


13 Option C The blood goes through heart only once in fishes.

14. Option B) 1 Guard cells, 2 Stoma, 3 Epidermal cells, 4 Chloroplast

15. Option is (c) (i), (ii) and (iii)


16. Option B Carbon dioxide given out by the germinating seeds will be absorbed by
the KOH.
17. Option (b) (i), (iii) and (iv)
18. Option (d) At less than F
Explanation: For virtual image object must lie between F and optical centre.
19. Option (c) At infinity
20. Option (d) All of the above
21. (B) MS

22. (D) Convex lens


23 (A) A convex lens has 4 dioptre powers having a focal length 0.25 m
24. (C) Concave, plane and convex
25. (A)Copper Sulphate (SECTION-B)
26. (B) Wash the hand immediately with plenty of water and apply a paste of sodium
hydrogen carbonate.
27. (A) --(a) & (b)Reaction of water with quick lime
Dilution of an acid
28. (B) Has low melting point
29. (A) An increase in the pH of solution
30. (B) Tartaric acid
31. B-when we add water in the acid it will be highly exothermic reaction which will
be dangerous so we must add acid in the water because it's not dangerous.
32. C- Copper is heated with the oxygen present in the air and forms a black-
coloured substance on its surface.
33. A- Due to entry of water into it, a positive pressure develops in a plant cell or
system. This positive hydrostatic pressure is also known as turgor pressure.
Turgor pressure keeps the cells and their organelles stretched; this is essential
for proper functioning of a cell. It gives support to non-woody tissues like
parenchyma. During growth, turgor pressure is essential for cell enlargement.
34. D-Refraction is the bending of light rays after entering a medium where its speed
is different. Due to refraction of light, when a ray of light passes from a rarer
medium to a denser medium, bends towards the normal to the boundary
between the two media.

35. A. (a) —> (iv) (b) —> (iii) (c) —> (ii) (d) —> (i)
36. B-As the heart pumps blood, a series of valves open and close tightly. These
valves ensure that blood flows in only one direction, preventing backflow.
- The tricuspid valve is situated between the right atrium and right ventricle.
- The pulmonary valve is between the right ventricle and the pulmonary artery.
- The mitral valve is between the left atrium and left ventricle.
The aortic valve is between the left ventricle and the aorta
37. C-Small intestine is largest part of alimentary canal.
38. C-The transpiration process, pulls water upwards with the help of-cohesion and
adhesion properties of water molecules. According to transpiration pull theory,
due to transpiration, the water column inside the plant comes under tension.
This is called 'transpiration Puli'. On account of this tension, the water column is
pulled up passively from below to top of the plant (almost like a rope). Root
pressure is the pressure that forces water, absorbed from the soil, to move
through the roots and up i.e., pushes it up) the stem of a plant. It may be due to
both the osmosis of water from the soil into the root cells, and the active
pumping of salts into xylem tissue which maintains a concentration gradient
along which the water moves
39. B-Concave lenses always produce images that are upright, virtual, reduced in
size, and located on the object's side of the lens. Therefore, the magnification
produced by a concave lens is always less than 1.
40. B-Given, height of object = 10 mm, height of image = 5 mm, image distance = -30
cm,
Magnification `=(h’)/h`
`=5/(10)=1/2=0.5`
Since, `m=v/u`
Or, `0.5=(-30text(cm))/u`
Or, `u=-(30)/(0.5)=-60` cm
Mirror formula `1/v+1/u=1/f`
Or, `-1/(60)-1/(30)=1/f`
Or, `(-1-2)/(60)=1/f`
Or, `1/f=-3/(60)=-1/(20)`
Or, `f=-20` cm

41. c) Kidney→ ureter → urinary bladder→ urethra


42. d) Right atrium → Right ventricle → Lungs → Left atrium → Left ventricle
43. b) Diopter
44. a) greater than unity.

45. (c) Atmospheric refraction


46. (b) B
47. (A)At infinity
48. (B) Na
49. (d) green (SECTION-C)

50. (b) Brown


51. (a) Decomposition reaction
52. (d) all of the above
53. (C) Sunlight
54. (A)Photosynthesis
55. (A)Due to starch
56. (B)Plant should have access to air, sunlight and water
57. (b) due to refraction
58. (D)No unit
59. (c) n = c/v
60. (b) greater

Answers of Questions in lieu of Diagram based questions:

1) (A) only(i)

5(B) Hydrogen

6(B) Aqueous solution of NaOH

9(B) Due to loss of water molecules from copper sulphate.

11(B) Nostrils- pharynx- larynx- trachea-alveoli


12(C)Amylase secreted by saliva, pepsin secreted by stomach, trypsin secreted by pancreas, lipase secreted by pancreas.

13(D) Transpiration

16(B) They have thin walls with valves inside, blood flows under high pressure and
carry blood away from the heart to various organs of the body
19(C)Refraction of sun light from water drops

21(D.) Behind the mirror.

22(B.)1

44(B.)40 cm

46(D) 3.5 cm & Erect


KENDRIYA VIDYALAYA SANGATHAN( RO JAIPUR)
MARKING SCHEME – SQP 3
CLASS:X
Science (086)

Q.NO. ANSWERS

Section - A
01 (d) Na2SO4, BaSO4
02 C
03 (c) 1 and 3

04 (d) Cu + 2HNO3 →Cu(NO3)2 + 2NO2 + H2O

05 (b) a reducing agent


06 (a) White
07 (a) Water < acetic acid < hydrochloric acid

08 (a) The formation of salt + water ( H2O)

09 (d) Carbon
10 (d) (i) and (iv)
11 (a) Mitochondria
12 (b) Starch breaking down into sugars

13
(b) Amoeba
14
(b) Emulsify fats
15
(d) Regulate exit of food
16
(b)Transpiration, photosynthesis and respiration

17 (a) concave lens


18 (c) between C and F
19 (a)The incident ray, reflected ray, and normal at the
point of reflection lie on a common plane

20 (d) change in the density of the medium

21 (c) 5 dioptre
22 ( b) Red
23 (d) Scattering of light
24 (c) Refraction of light
Section – B
25 (a) (i) and (ii) Reason: In both the cases,heat energy
is evolved.
26 (a) X – H2O , Y – NaOH

27 (d) Al2O3
28 (a) CaSO4.2H2O and CaSO4.1/2 H2O

29 (a) Baking soda

30 (a) Lead
31 (a) Both A and R are true and R is the correct
explanation of A

32 (c) A is true but R is false


33 (b) Both A and R are true and R is not the correct
explanation of A

34 (c) Assertion is true but Reason is false.

35 (b) Both A and R are true and R is not the correct


explanation of A

36 (c) NaAlO2
37
(c) Left atrium
38
(b) Small intestine
39
(b) (i),(ii) & (iii)
40 (b) Dialysis
41 ( b) Digest protein.
42 (a) 0.5 times

43 (b) Q
44 (d) Glycerine
45 (b) B
46 (b) Concave
47 (d) between focus and centre of curvature

48 (d)vacuum
Section – C
49 (d) All of these

50 (d) All of these

51 (b) Gives chlorine on exposure to atmosphere

52 (a) CaOCl2

53 (c) Ureter
54 (c) Nephron
55 (c) Dialysis
56 (d) Urinary bladder
57 (b) The bending of pencil.
58 (d) None of these
59 (d) both (b) and (c)
60 (c)1.32
KENDRIYA VIDYALAYA SANGATHAN( RO JAIPUR)
MARKING SCHEME – SQP 4
CLASS:X
Science (086)
Q.No Answers

Section-A

1 (d) Electric current

2 (c) The reaction is an example of a combination reaction as two compounds


react to form a single compound

3 (a) Basic because it turns red litmus to blue

4 (b) Bromine

5 (a) 18

6 (a) Methanoic acid

7 (b) HCl

8 (d) Both (b) and (c)

9 (c) Green

10 (b) Baking soda

11 (c) Potassium hydroxide

12 (b) take the urine from the kidney to the urinary bladder

13 (b) Guard Cells

14 (d) Amylase Emulsify fats

15 (b) Spongy

16 (a) Oxygenated blood

17 (d) Dioptre

18 (b) Focus

19 (b) At 2F

20 (c) unit less

21 (a) Greater than unity

22 (b) Decreases velocity


23 (c) 50cm

24 (b) Atmospheric Refraction of light by different layers of varying refractive


indices

Section-B

25 ( b) Remove the layer of magnesium oxide from the ribbon surface

26 ( a) Melting of candle wax when heated

27 (a)Mg > Zn > Fe

28 (a) CaO

29 (b) It catches fire and forms hydroxide

30 (a) 0°

31 (c) Assertion (A) is true but reason (R) is false.

32 (a)Both assertion (A) and reason (R) are true and reason (R) is the correct
explanation of assertion.

33 (a)Both assertion (A) and reason (R) are true and reason (R) is the correct
explanation of assertion.

34 (a)Both assertion (A) and reason (R) are true and reason (R) is the correct
explanation of assertion.

35 (a)CuSO4 + Fe

36 (d)Excretion

37 1 and 2

38 (a) Translocation

39 ( a) Concave mirror

40 (d) A concave lens has -4 dioptre power having a focal length -0.25 m

41 (c) Iodine

42 (d) All of these

43 (c) 1,9

44 (b) There is no atmosphere at great heights

45 Scattering due to dust particles and air molecules causes this phenomenon

46 (d) Dispersion

47 purple
48 Is scattered the least by smoke or fog

Section -C

49 Potassium

50 Iodine

51 Water

52 Formation of ionic bonds involves sharing of electrons

53 (a) It is similar to the plasma of blood, but is colourless and contains fewer
proteins.

54 (a) intercellular spaces

55 (b)return interstitial fluid to blood

56 forming RBCs

57 (b) 10 cm

58 (d) None of these

59 (a) A parallel beam of light

60 (c) R

Questions in lieu of diagram based questions for VI candidates

Total Alternative Questions -04

21 (a) Greater than unity

21 (a) Greater than unity

21 (a) Greater than unity

43 (c) 1,9
KENDRIYA VIDYALAYA SANGATHAN( RO JAIPUR)
MARKING SCHEME – SQP 5
CLASS:X
Science (086)
Q.NO. ANS.
1 C- Sodium chloride
2 C- Carbon dioxide
3 D- A, B and C
4 D- CaSO4
5 C-C
6 B-Light
7 A- Exothermic reaction
8 B- inversely proportional to pH
9 C- Neutralization
10 A- Copper oxide
11 B- The amount of dissolved oxygen in water is fairly low as compared to the
amount of oxygen in the air
12 B-Amylase, pepsin, trypsin
13 C- Both A and B are correct
14 C-(ii) and (iv)
15 B- A-Vein, B-Artery, C-Vein, D-Artery
16 A- Starch breaking down into sugars
17 B- a convex mirror
18 B- Converging
19 D- all mirrors irrespective of their shape
20 B- Air
21 B-10CM
22 D= +0.5
23 B- wavelength and speed
24 A- blue light is dispersed more than red light
25 A- Exothermic process
26 A- The litmus paper used is dry
27 D- Sodium > Magnesium > Zinc > Iron
28 A- Ductility
29 D-(ii) and (iv)
30 C-(ii) only
31 C- A is true but R is false
32 C- A is true but R is false
33 D- A is False but R is true
34 D- A is False but R is true
35 D-(ii) and (iv)
36 B- Hippocampus, Exocoetus, Anabas
37 A- Nostril → Trachea → Bronchioles → Alveoli
38 D- Both a and b
39 B-30 cm in front of the mirror
40 A- is less than one
41 B- Parasitic nutrition
42 B- Mouth → oesophagus → stomach → small intestine → large intestine
43 C- between C and F
44 A- difference in optical density
45 D- All
46 A- virtual
47 A-8 cm
48 A-Sonorousness
49 A- pH of P increases while that of Q decreases
50 A- Formic acid
51 C- it contains sodium hydroxide which neutralizes the effect of formic acid
52 B- Y only
53 C- Temporarily store urine
54 B-Urea
55 C- Excretion
56 B-Kidneys
57 B- Centre
58 A- Centre
59 A- Positive
60 C-1 D

MARKING SCHEME OF
Questions in lieu of diagram based questions for VI candidates
Total alternative questions -26
Q.NO. ANS.
2 B- Turn white
3 D- Iodine
5 C- Iodine
11 A- food vacuole
12 C- starch
14 C- Valves in heart
15 D- Small intestine
16 D- conversion of pyruvate to lactic acid
18 B- Convex mirror
22 A- Height of image/ Height of object

23 D- All
24 C- Blue,pink,Green
26 B- Baking soda
27 D-(iii) and (iv)
30 D- Gold
44 C- Between focus and pole
45 D-1.80
47 A- Concave
53 B- Yeast, mushroom, bread mould
54 B- Taenia
55 B-Mushroom
56 C- Utilization of energy obtained by plants
57 B-Red
58 D-Yellow
59 D- Orange, yellow, Blue
60 B- The splitting of a beam of light into component colours

You might also like